Nurs 1600 Fluids & electrolytes

Ace your homework & exams now with Quizwiz!

A patient with severe vomiting and diarrhea was brought to the emergency department in a semiconscious condition. What interventions does the nurse implement to stabilize the patient? Select all that apply. a. Monitor the urine output. b. Monitor the respiratory rate. c. Administer the oral rehydration salts. d. Monitor blood pressure and heart rate. e. Administer replacement fluids intravenously. f. Administer drugs to correct the cause of dehydration.

e, f Nursing priorities in stabilizing a patient with dehydration include patient safety, fluid replacement, and drug therapy. To increase the fluid volume to normal, intravenous replacement fluids are administered. Drug therapy can correct some causes of dehydration such as diarrhea and vomiting. Therefore antidiarrheals and antiemetic drugs are administered to manage dehydration in the patient. Monitoring the urine output, respiratory rate, blood pressure, and heart rate is necessary to prevent further complications. However, monitoring these parameters will not stabilize the patient with dehydration. The patient cannot tolerate oral fluids when the dehydration is severe. Therefore oral rehydration salts are not preferred initially to stabilize the patient.

Which age-related change to the endocrine gland affects fluid balance in the body? A. Adrenal atrophy B. Insulin resistance C. Increased secretion of aldosterone D. Decreased secretion of antidiuretic hormone

A An age-related change to the endocrine gland is adrenal atrophy, which causes poor regulation of sodium and potassium, placing the patient at risk for fluid imbalance. Insulin resistance is not an age-related change to the endocrine gland. Although increased secretion of aldosterone and decreased secretion of antidiuretic hormone can impact fluid balance, these are not caused by age-related changes to the endocrine gland.

The nurse is providing care to a patient who is admitted with fluid volume overload. Which electrolyte imbalances does the nurse anticipate for this patient based on the admitting diagnosis? Select all that apply. A. Hypokalemia B. Hyponatremia C. Hypercalcemia D. Hypochloremia E. Hypermagnesemia

A, B, D A patient admitted with fluid volume overload will often experience hyponatremia, hypokalemia, and hypocholoremia. Hypercalcemia and hypermagnesemia are not anticipated electrolyte imbalances associated with fluid volume overload.

A patient's potassium level is 3.0. Which foods would you encourage the patient to consume?* A. Cheese, collard greens, and fish B. Avocados, strawberries, and potatoes C. Tofu, oatmeal, and peas D. Peanuts, bread, and corn

B

Select below the CORRECT sequence in how the renin-angiotensin-aldosterone system works:* A. Angiotensin I -> Angiotensin II -> ACE -> Renin -> Angiotensinogen B. Renin-> Angiotensinogen -> Angiotensin I -> ACE -> Angiotensin II C. Renin -> Angiotensin I -> Angiotensinogen -> ACE -> Angiotensin II D. Angiotensinogen -> Renin -> ACE -> Angiotensin I -> Angiotensin II

B

What is the role of aldosterone?* A. It causes constriction of vessels. B. It causes the kidneys to keep sodium and water. C. It causes the kidneys to keep potassium and water. D. It causes the kidneys to only keep water.

B

A patient with hypovolemic hyponatremia is started on IV fluids. Which of the following fluids do you expect the patient to be started on? A. 0.45% Saline B. 3% Saline C. D5W D. 0.33% Saline

B 3% Saline Patients with hypovolemic hyponatremia are started on a hypertonic solution (the circulatory system is dehydrated & and the cells are swollen...so a hypertonic solution will shrink the cells and increase fluid volume) and 3% Saline is the only hypertonic solutions. The other options are either hypotonic or isotonic.

The nurse is planning care for a client with fluid volume excess and hyponatremia. Which intervention will the nurse include? a. administer NS at keep open rate b. limit fluids to 1000 mL/day c. Administer NS iv at 125mL/hr continuously d. administer furosemide 80mg slow IVP

B By limiting fluid intake to 1000 mL/day the client's body will be given a chance to remove some of the excess fluid volume. Due to the hyponatremia, sodium level should not be lowered and an increased sodium intake could cause an even greater fluid volume excess. Infusions of fluids will increase the fluid volume excess. Diuretics will cause a further decline in sodium levels.

How much fluid should a healthy adult consume each day to maintain adequate urine output? A. 2 L/day B. 2.3 L/day C. 2.6 L/day D. 2.9 L/day

B Most healthy adults take in about 2300 mL of fluid each day, which is equivalent to 2.3 L/day. The other figures do not accurately account for the amount of daily fluid from food and liquids.

The nurse selects the problem of "fluid volume excess" for a patient. Which intervention should be included in the plan of care? a. change the IV fluid from 0.9% NS to Dextrose 5% and water b. restrict the amount of sodium in the diet c. monitor blood glucose levels d. prepare the client for hemodialysis

B Restricting the sodium in the patient's diet to be between 2 and 4 grams a day can help to reduce fluid retention. Sodium binds with water in the body to retain fluid both inside and outside the cells. A healthcare provider's prescription is needed to change the type of IV fluid. There is no reason to monitor blood glucose levels. Hemodialysis is not indicated.

The nurse is providing education to a patient diagnosed with hypertension. Which statement by the nurse is most appropriate to help the patient maintain a normal fluid balance? A. "Increase your intake of water each day to increase urine output." B. "Limit your intake of sodium to decrease the water you are retaining." C. "Foods rich in potassium, such as bananas, will increase urine output." D. "Foods rich in calcium, such as milk, will help to decrease urine output."

B The best way for a patient with hypertension to maintain a normal fluid balance is to limit the intake of dietary salt. The reason for this is that a high sodium intake raises the blood level of sodium, causing more water to be retained in the blood volume and raising blood pressure. The patient may be asked to decrease their fluid intake due to edema. Foods rich in potassium and calcium will not help the patient with hypertension maintain a normal fluid balance.

What is the neurologic impact of age-related changes on fluid balance? A. Diminished knee reflex B. Diminished thirst reflex C. Diminished cough reflex D. Diminished baroreceptor reflex

B The neurologic impact of a person's age on fluid balance is a diminished thirst reflex, which results in decreased fluid intake, thereby increasing the risk of dehydration. Diminished knee reflexes, diminished cough reflex, and diminished baroreceptor reflexes are not associated with changes in fluid balance.

A client without health problems is being treated for cellulitis of the hand. The client's serum potassium level upon admission was 4.5 mEq/L however the most recent level is 7 mEq/L. Which action should the nurse take? a. Notify the healthcare provider of results b. question the results and have the specimen redrawn c. encourage the client to increase the intake of bananas d. initiate seizure precautions

B client who has no health problems being treated for cellulitis should have an elevated serum potassium level questioned. When the serum potassium level increases from 4.5 mEq/L to 7.0 mEq/L with no risk factors for hyperkalemia, false high results should be suspected because of hemolysis of the specimen. The healthcare provider would likely question results also. Bananas are a food high in potassium. Seizures are not a clinical manifestation of hyperkalemia.

After teaching a client who is being treated for dehydration, a nurse assesses the clients understanding. Which statement indicates the client correctly understood the teaching? a. I must drink a quart of water or other liquid each day. b. I will weigh myself each morning before I eat or drink. c. I will use a salt substitute when making and eating my meals. d. I will not drink liquids after 6 PM so I wont have to get up at night.

B One liter of water weighs 1 kg; therefore, a change in body weight is a good measure of excess fluid loss or fluid retention. Weight loss greater than 0.5 lb daily is indicative of excessive fluid loss. The other statements are not indicative of practices that will prevent dehydration.

A nurse is caring for a client who has a serum calcium level of 14 mg/dL. Which provider order should the nurse implement first? a. Encourage oral fluid intake. b. Connect the client to a cardiac monitor. c. Assess urinary output. d. Administer oral calcitonin (Calcimar).

B This client has hypercalcemia. Elevated serum calcium levels can decrease cardiac output and cause cardiac dysrhythmias. Connecting the client to a cardiac monitor is a priority to assess for lethal cardiac changes. Encouraging oral fluids, assessing urine output, and administering calcitonin are treatments for hypercalcemia, but are not the highest priority.

What history and assessment findings may be associated with hypocalcemia in a 22-year-old man? Select all that apply. A. Absent bowel sounds B. Tingling around the mouth C. Awakening at night with muscle spasms in the calf D. Decreased deep tendon reflexes without paresthesia E. Recent blunt trauma to the throat during a football game

B, C, E A history of anterior neck injury may be associated with hypocalcemia. Symptoms of hypocalcemia include "charley horses" in the calf during rest or sleep, and tingling in the lips. Hypocalcemia does not affect bowel sounds. Decreased deep tendon reflexes without paresthesia is a neuromuscular change in hypercalcemia.

Which foods will the nurse teach a patient with hyperkalemia to avoid? Select all that apply. A. Sugar B. Oranges C. Broccoli D. Cranberries E. Dairy products

B, C, E A patient with hyperkalemia has high levels of potassium, so the nurse should instruct the patient to avoid oranges, broccoli, and dairy products, which are all high in potassium. Sugar is low in potassium, so this patient does not necessarily need to avoid it in order to manage his or her hyperkalemia, but he or she should limit sugar consumption to healthy levels. Cranberries are low in potassium, so telling the patient to avoid cranberries is unnecessary.

The nurse is administering sodium chloride 0.9% (normal saline) intravenously to a patient who is dehydrated. Which assessments does the nurse perform to evaluate the effectiveness of rehydration therapy? Select all that apply. A. Temperature B Urinary output C. Bowel sounds D. Pulse rate and quality E. Level of consciousness

B, D Pulse rate and quality as well as urinary output best reflect improving volume status with rehydration therapy. Temperature, level of consciousness, and bowel sounds are not indicators of an improving volume state, although elevated temperature can worsen dehydration.

A nurse develops a plan of care for a client who has a history of hypocalcemia. What interventions should the nurse include in this clients care plan? (Select all that apply.) a. Encourage oral fluid intake of at least 2 L/day. b. Use a draw sheet to reposition the client in bed. c. Strain all urine output and assess for urinary stones. d. Provide nonslip footwear for the client to use when out of bed. e. Rotate the client from side to side every 2 hours.

B, D Clients with long-standing hypocalcemia have brittle bones that may fracture easily. Safety needs are a priority. Nursing staff should use a draw sheet when repositioning the client in bed and have the client wear nonslip footwear when out of bed to prevent fractures and falls. The other interventions would not provide safety for this client.

Which hormones play a role in the regulation of sodium balance by the kidneys? Select all that apply. A. Cortisol B. Aldosterone C. Angiotensin D. Natriuretic peptide (NP) E. Antidiuretic hormone (ADH)

B, D, E Serum sodium levels are regulated by the kidneys under the influence of aldosterone, natriuretic peptide (NP), and antidiuretic hormone (ADH). Low serum sodium levels inhibit the secretion of ADH and NP and trigger the secretion of aldosterone. This increases the serum sodium levels by increasing the reabsorption of sodium and enhancing water loss by the kidney. High serum sodium levels inhibit aldosterone secretion and stimulate the secretion of ADH and NP. These hormones increase the excretion of sodium and reabsorption of water by the kidney. Cortisol and angiotensin do not regulate the serum sodium levels.

A nurse assesses a client who is prescribed a medication that inhibits aldosterone secretion and release. For which potential complications should the nurse assess? (Select all that apply.) a. Urine output of 25 mL/hr b. Serum potassium level of 5.4 mEq/L c. Urine specific gravity of 1.02 g/mL d. Serum sodium level of 128 mEq/L e. Blood osmolality of 250 mOsm/L

B, E Aldosterone is a naturally occurring hormone of the mineralocorticoid type that increases the reabsorption of water and sodium in the kidney at the same time that it promotes excretion of potassium. Any drug or condition that disrupts aldosterone secretion or release increases the clients risk for excessive water loss (increased urine output), increased potassium reabsorption, decreased blood osmolality, and increased urine specific gravity. The client would not be at risk for sodium imbalance.

A patient has a sodium level of 119. Which of the following is NOT related to this finding? A. Over secretion of ADH (antidiuretic hormone) B. Low salt diet C. Inadequate water intake D. Hypotonic fluid infusion (overload)

C

A patient is presenting with an orthostatic blood pressure of 80/40 when she stands up, thready and weak pulse of 58, and shallow respirations. In addition, the patient has been having frequent episodes of vomiting and nausea and is taking hydrochlorothiazide. Which of the following findings would explain the patient's condition?* A. Potassium level of 7.0 B. Potassium level of 3.5 C. Potassium level of 2.4 D. None of the options are correct

C

During RAAS activation, what gland releases aldosterone?* A. Hypothalamus B. Thymus C. Adrenal cortex D. Pituitary

C

What component of the RAAS (renin-angiotensin-aldosterone system) is created and found in the liver that is activated by renin?*A. Aldosterone B. Angiotensin I C. Angiotensinogen D. Angiotensin II

C

What is the goal of the renin-angiotensin-aldosterone system (RAAS)?* A. Decrease blood pressure B. Increase the heart rate C. Increase the blood pressure D. Decrease the heart rate

C

Which patient is at risk for hyperkalemia?* A. Patient with Parathyroid cancer B. Patient with Cushing's Syndrome C. Patient with Addison's Disease D. Patient with breast cancer

C

You have completed diet teaching with a patient who has hypernatremia. Which statement by the patient causes concern? A. "I will buy fresh vegetables and fruits." B. "I will avoid eating canned foods." C. "I'm glad I can still eat sandwiches with bologna." D. "I will avoid cooking with butter."

C

A client with renal failure seeks medical attention after missing three dialysis treatments.Which set of arterial blood gases (ABGs) should indicate to the nurse that the client is in metabolic acidosis? a. PH of 7.43, PCO2 of 36, HCO3 of 26 b. PH of 7.41, PCO2 of 49, HCO3 of 30 c. PH of 7.33, PCO2 of 35, HCO3 of 17 d. PH of 7.25, PCO2 of 56, HCO3 of 28

C Evidence of acidosis includes a pH of 7.33, PC02 of 35, and HC03 of 17 and a pH of 7.25, PC02 of 56, and HC03 of 28. However the pH of 7.25 indicates respiratory acidosis. The pH of 7.41 indicates compensated metabolic alkalosis and a pH of 7.43 would be considered normal.

A nurse cares for a client who has a serum potassium of 7.5 mEq/L and is exhibiting cardiovascular changes. Which prescription should the nurse implement first? a. Prepare to administer sodium polystyrene sulfate (Kayexalate) 15 g by mouth. b. Provide a heart healthy, low-potassium diet. c. Prepare to administer dextrose 20% and 10 units of regular insulin IV push. d. Prepare the client for hemodialysis treatment.

C A client with a high serum potassium level and cardiac changes should be treated immediately to reduce the extracellular potassium level. Potassium movement into the cells is enhanced by insulin by increasing the activity of sodium-potassium pumps. Insulin will decrease both serum potassium and glucose levels and therefore should be administered with dextrose to prevent hypoglycemia. Kayexalate may be ordered, but this therapy may take hours to reduce potassium levels. Dialysis may also be needed, but this treatment will take much longer to implement and is not the first prescription the nurse should implement. Decreasing potassium intake may help prevent hyperkalemia in the future but will not decrease the clients current potassium level.

After teaching a client to increase dietary potassium intake, a nurse assesses the clients understanding. Which dietary meal selection indicates the client correctly understands the teaching? a. Toasted English muffin with butter and blueberry jam, and tea with sugar b. Two scrambled eggs, a slice of white toast, and a half cup of strawberries c. Sausage, one slice of whole wheat toast, half cup of raisins, and a glass of milk d. Bowl of oatmeal with brown sugar, a half cup of sliced peaches, and coffee

C Meat, dairy products, and dried fruit have high concentrations of potassium. Eggs, breads, cereals, sugar, and some fruits (berries, peaches) are low in potassium. The menu selection of sausage, toast, raisins, and milk has the greatest number of items with higher potassium content.

A nurse teaches a client who is at risk for mild hypernatremia. Which statement should the nurse include in this clients teaching? a. Weigh yourself every morning and every night. b. Check your radial pulse twice a day. c. Read food labels to determine sodium content. d. Bake or grill the meat rather than frying it.

C Most prepackaged foods have a high sodium content. Teaching clients how to read labels and calculate the sodium content of food can help them adhere to prescribed sodium restrictions and can prevent hypernatremia. Daily self-weighing and pulse checking are methods of identifying manifestations of hypernatremia, but they do not prevent it. The addition of substances during cooking, not the method of cooking, increases the sodium content of a meal.

A client with hyperkalemia is prescribed sodium polystyrene. Which will the nurse monitor after giving the client the medication? a. urine output b. respiratory rate c. bowel movements d. tall peaked T waves on cardiac telemetry

C Sodium polystyrene or Kayexalate causes potassium to be exchanged for sodium in the intestines and excreted through bowel movements. If the client does not have stools, the drug cannot work properly. Sodium polystyrene does not affect urine output, respiratory rate, or cardiac rhythm.

A client at risk for developing hyperkalemia states, I love fruit and usually eat it every day, but now I cant because of my high potassium level. How should the nurse respond? a. Potatoes and avocados can be substituted for fruit. b. If you cook the fruit, the amount of potassium will be lower. c. Berries, cherries, apples, and peaches are low in potassium. d. You are correct. Fruit is very high in potassium.

C Not all fruit is potassium rich. Fruits that are relatively low in potassium and can be included in the diet include apples, apricots, berries, cherries, grapefruit, peaches, and pineapples. Fruits high in potassium include bananas, kiwi, cantaloupe, oranges, and dried fruit. Cooking fruit does not alter its potassium content.

The nurse is caring for a group of clients. Which clients would the nurse identify as being at risk for developing hypercalcemia? Select all that apply. a. The client who has a positive Chvostek's sign b. The client who has steatorrhea c. the client with a malignancy d. the client who overuses antacids

C, D Clients with a malignancy are likely to have hypercalcemia due to the excessive secretion of parathyroid hormone. Malignancy-induced hypercalcemia is also termed humoral hypercalcemia. Many antacids are calcium salts, and overusing them can increase calcium levels to cause hypercalcemia. A positive Chvostek's sign is associated with hypocalcemia. Steatorrhea is the inability to digest fat.

After administering 40 mEq of potassium chloride, a nurse evaluates the clients response. Which manifestations indicate that treatment is improving the clients hypokalemia? (Select all that apply.) a. Respiratory rate of 8 breaths/min b. Absent deep tendon reflexes c. Strong productive cough d. Active bowel sounds e. U waves present on the electrocardiogram (ECG)

C, D A strong, productive cough indicates an increase in muscle strength and improved potassium imbalance. Active bowel sounds also indicate treatment is working. A respiratory rate of 8 breaths/min, absent deep tendon reflexes, and U waves present on the ECG are all manifestations of hypokalemia and do not demonstrate that treatment is working.

The nurse manager of the medical-surgical unit assigns which patient to the LPN/LVN? A. 44-year-old admitted with dehydration who has a heart rate of 126 B. 54-year-old just admitted with hyperkalemia who takes a potassium-sparing diuretic at home C. 64-year-old admitted yesterday with heart failure who still has dependent pedal edema D. 74-year-old who has just been admitted with severe nausea, vomiting, and diarrhea

C. Because the patient with heart failure is the most stable of the four patients, this patient is most appropriate to assign to the LPN/LVN. Dehydration, tachycardia, potassium overload, and gastrointestinal signs and symptoms in a patient indicate that he or she is unstable and should be cared for by RN staff members.

Tall peaked T-waves, flat P-waves, prolonged PR intervals and widened QRS complexes can present in which of the following conditions?* A. Hypocalemia B. Hypercalemia C. Hypokalemia D. Hyperkalemia

D

What is the role of ACE in the renin-angiotensin-aldosterone system?* A. It activates angiotensinogen which turns into angiotensin I. B. It causes the kidney cells to release renin. C. It causes angiotensin II to trigger the adrenal glands to produce aldosterone. D. It converts angiotensin I into angiotensin II.

D

When the RAAS is activated due to a change in body hemodynamics, the __________________ stimulates the ___________________ cells in the kidneys to release _______________.* A. parasympathetic nervous system; mesangial; aldosterone B. sympathetic nervous system; podocytes; renin C. parasympathetic nervous system; juxtaglomerular; aldosterone D. sympathetic nervous system; juxtaglomerular; renin

D

Which of the following is indicative of an EKG change in a case of hypokalemia?* A. Widened QRS complex and prolonged PR interval B. Prolonged ST interval and Widened T-wave C. Tall T-waves and depressed ST segment D. ST depression and inverted T-wave

D

Which newly written prescription does the nurse administer first? A. Oral calcium supplements to a patient with severe osteoporosis B. Oral phosphorus supplements to a patient with acute hypophosphatemia C. Intravenous (IV) normal saline to a patient with a serum sodium of 132 mEq/L D. Oral potassium chloride (KCl) to a patient whose serum potassium is 3 mEq/L

D Because minor changes in serum potassium level can cause life-threatening dysrhythmias, the first priority should be to administer potassium supplements to the patient with hypokalemia. The electrolyte disturbance (sodium level of 132 and low phosphorus level) and the need for calcium in this patient are not immediately life-threatening.

A client who is prescribed bicarbonate for metabolic acidosis also takes a fluoroquinolone. For which interaction should the nurse monitor this client? a. urine retention b. abdominal discomfort c. renal failure d. increased risk for crystalluria

D Bicarbonate may increase the risk of a renal disorder. The crystals indicate cystinuria, which places the client at risk for cystine calculi. Bicarbonate taken with a fluoroquinolone does not cause urinary retention, abdominal discomfort, or renal failure.

What is the major cause of death in patients diagnosed with hypokalemia? A. Stroke B. Renal failure C. Cardiac arrest D. Respiratory insufficiency

D Respiratory changes may occur in patients with hypokalemia because of respiratory muscle weakness resulting in shallow respirations. The respiratory status of a patient with hypokalemia should be assessed at least every 2 hours because respiratory insufficiency is the major cause of death for these patients. A stroke is not a risk factor for hypokalemia. Hypokalemia does not cause renal failure; rather, hyperkalemia is caused by renal failure. Dysrhythmias may occur due to hypokalemia but are not the major cause of death in patients with hypokalemia.

Which value falls within the range for the minimal amount of urine that needs to be produced in 24 hours? a. 150 mL/day b. 250 mL/day c. 350 mL/day D. 450 mL/day

D The minimum volume of urine per day needed to excrete toxic waste products is 400 to 600 mL and is called obligatory urine output. Urine output between 100 and 400 mL per day is called oliguria or hypouresis.

A patient presents to the emergency room with confusion, altered level of consciousness, dry mouth, scaly skin, and very loose-fitting shoes. Which finding in the patient's history is significant? A. Unusual increase in body weight B. Decreased exposure to higher altitudes C. Decreased sense of thirst in the patient D. Decreased ingestion of solid foods such as gelatin

D The patient whose shoes are suddenly loose may be dehydrated. Dehydration is manifested as changes in mental status such as confusion and altered levels of consciousness, dry mouth, scaly skin, and an elevated serum osmolarity. If the patient stopped eating clear liquid foods, such as gelatin, there may be concerns about dehydration. Weight gain that is proportional to fluid loss is not an indication of dehydration. Patients with decreased exposure to higher altitudes are at a lower risk of dehydration. A decreased sense of thirst in patients may not be a sign of dehydration.

A patient with mild hypokalemia caused by diuretic use is discharged home. The home health nurse delegates which of these interventions to the home health aide? A. Instruction on the proper use of drugs B. Education about potassium-rich foods C Assessment of muscle tone and strength D. Measurement of the patient's urine output

D A home health aide may measure the patient's intake and output, which then would be reported to the RN. Assessment, education, and instruction are higher-level nursing actions that should be done by the RN.

A nurse is caring for an older adult client who is admitted with moderate dehydration. Which intervention should the nurse implement to prevent injury while in the hospital? a. Ask family members to speak quietly to keep the client calm. b. Assess urine color, amount, and specific gravity each day. c. Encourage the client to drink at least 1 liter of fluids each shift. d. Dangle the client on the bedside before ambulating.

D An older adult with moderate dehydration may experience orthostatic hypotension. The client should dangle on the bedside before ambulating. Although dehydration in an older adult may cause confusion, speaking quietly will not help the client remain calm or decrease confusion. Assessing the clients urine may assist with the diagnosis of dehydration but would not prevent injury. Clients are encouraged to drink fluids, but 1 liter of fluid each shift for an older adult may cause respiratory distress and symptoms of fluid overload, especially if the client has heart failure or renal insufficiency.

After teaching a client who is prescribed a restricted sodium diet, a nurse assesses the clients understanding. Which food choice for lunch indicates the client correctly understood the teaching? a. Slices of smoked ham with potato salad b. Bowl of tomato soup with a grilled cheese sandwich c. Salami and cheese on whole wheat crackers d. Grilled chicken breast with glazed carrots

D Clients on restricted sodium diets generally should avoid processed, smoked, and pickled foods and those with sauces and other condiments. Foods lowest in sodium include fish, poultry, and fresh produce. The ham, tomato soup, salami, and crackers are often high in sodium.

Which fruit will the nurse remove from the dietary tray of a patient with high potassium levels? a. Kiwi b. Berries c. Apricots d. Grapefruit

a Kiwis are high in potassium, so the nurse will remove this food from the tray. However, berries, apricots, and grapefruits are low in potassium, so this patient does not need to avoid them.

The RN is caring for a patient who is severely dehydrated. Which nursing action can be delegated to unlicensed assistive personnel (UAP)? a. Providing oral care every 1 to 2 hours b. Infusing 500 mL of normal saline over 60 minutes c. Consulting with a health care provider about a patient's lab results d. Monitoring intravenous (IV) fluid to maintain the drip rate at 75 mL/hr

a.

The nurse is reviewing serum electrolytes and blood chemistry for a newly admitted patient. Which result causes the greatest concern? a. Glucose: 97 mg/dL b. Sodium: 143 mEq/L c. Potassium: 5.9 mEq/L d. Magnesium: 2.1 mEq/L

c. A potassium value of 5.9 mEq/L is high, and the patient should be assessed further. A glucose value of 97 mg/dL, a magnesium value of 2.1 mEq/L, and a sodium value of 143 mEq/L are normal values.

Select all the roles of angiotensin II when it is activated in the renin-angiotensin-aldosterone system:* A. Activates bradykinin B. Triggers the release of aldosterone C. Increases the blood volume D. Causes vasodilation E. Increases systemic vascular resistance F. Causes the release of ADH (antidiuretic hormone)

B, C, F

The nurse, caring for a group of clients, notes that one client has a serum potassium level of 5.5 mEq/L. Which client should the nurse suspect has this level? a. the client with colitis b. the client with Cushing's syndrome c. the client who has been overusing laxatives d. the client who has sustained a traumatic burn

D Clients in the early stages of massive cell destruction, such as with heavy burns, are at risk for hyperkalemia. A potassium level greater than 5.1mEq/L is an indication of hyperkalemia. The clients with Cushing's syndrome, colitis, and laxative overuse are at risk for hypokalemia.

Which patient below is considered hypernatremic? A. A patient with a sodium level of 155 B. A patient with a sodium level of 145 C. A patient with a sodium level of 120 D. A patient with a sodium level of 136

a

What is the most appropriate nursing intervention in a patient with fluid overload and edema to prevent skin breakdown on pressure areas? a. Monitoring the urine output b. Changing position every 2 hours c. Administering diuretic drug therapy d. Limiting the intake of fluids and sodium

b Changing the patient's position every 2 hours may help in promoting circulation and preventing pressure on bony prominences. Monitoring the urine output helps to determine the effectiveness of therapies used to restore normal fluid balance. Administering diuretic drug therapy is an intervention to decrease fluid overload. Limiting the intake of fluids and sodium can decrease the fluid overload in chronic management.

Which two electrolyte imbalances does the nurse suspect based on the following assessment data of a patient? Physical: Hyperactive DTR Numbness/tingling in hands/feet Painful muscle contractions Peripheral pulse is weak, thready Psychosocial Assessment: Depression Confusion Nutrition History: Egg Allergy Lactose Intolerance Vegan Diet a. Hypercalcemia and hypernatremia b. Hypokalemia and hypermagnesemia c. Hypomagnesemia and hypocalcemia d. Hyperphosphatemia and hypercalcemia

c. Electrolyte imbalances frequently occur concurrently. The assessment data are consistent with hypomagnesemia and hypocalcemia. The patient's nutrition history also supports nutrient intake that is insufficient in calcium (i.e., dairy products).

A patient has a potassium level of 9.0. Which nursing intervention is priority?* A. Prepare the patient for dialysis and place the patient on a cardiac monitor B. Administer Spironolactone C. Place patient on a potassium restrictive diet D. Administer a laxative

A

A patient has a sodium level of 130. What is this condition called? A. Hyponatremia B. Hypernatremia C. Normal Sodium Level D. Hypercalcemia

A

A patient with nasogastric suctioning is experiencing diarrhea. The patient is ordered a morning dose of Lasix 20mg IV. Patient's potassium level is 3.0. What is your next nursing intervention?* A. Hold the dose of Lasix and notify the doctor for further orders B. Administered the Lasix and notify the doctor for further orders C. Turn off the nasogastric suctioning and administered a laxative D. No intervention is need the potassium level is within normal range

A

During the renin-angiotensin-aldosterone system activation, what gland releases antidiuretic hormone (ADH)?* A. Pituitary B. Thyroid C. Hypothalamus D. Adrenal cortex

A

What is the role of the antidiuretic hormone during RAAS?* A. Causes the kidneys to keep water B. Leads to vasoconstriction of vessels C. Activates the release of angiotensin I D. Prevents the activation of the parasympathetic nervous system

A

Which patient is at a potential risk for Digoxin toxicity?* A. A patient with Cushing's syndrome taking Laxis 20 mg IV twice a day B. A patient with a calcium level of 8.9 C. A patient with a potassium level of 3.8 D. A patient presenting with painful muscle spasms and positive Trousseau's sign

A

The nurse is providing care to a patient who is receiving diuretic therapy. Which electrolyte imbalance will the nurse monitor this patient for while providing care? A. Hypokalemia B. Hypercalcemia C. Hypernatremia D. Hypophosphatemia

A A patient who is prescribed diuretic therapy will require monitoring for hypokalemia. The nurse would not anticipate hypercalcemia, hypernatremia, or hypophosphatemia for this patient.

A patient has a serum magnesium level of 1.2 mEq/L. Which instruction by the nurse is appropriate? A "Notify me if you have diarrhea." B. "I will be giving you an intramuscular injection of magnesium sulfate." C. "Your urine may become darker and more concentrated in appearance." D. "I will be administering a phosphorus supplement in addition to the magnesium supplement."

A A serum magnesium level of 1.2 mEq/L represents mild hypomagnesemia (normal is 1.3-2.1 mEq/L) for which oral magnesium supplements are administered. Oral magnesium may lead to diarrhea; diarrhea contributes to magnesium loss. With severe hypomagnesemia, the IV route (instead of IM) is used because the IM route causes tissue damage and pain. Any medications containing phosphorus are contraindicated because they would contribute to associated hypocalcemia. Magnesium therapy does not affect the urine.

Which nursing action is recommended when providing care to a patient with hypokalemia? A. Question the continued administration of bumetanide. B. Administer prescribed oral potassium chloride before a meal or snack. C. Establish a peripheral IV, preferably in the hand, for administering IV potassium chloride. D. Obtain the prescribed vial of IV potassium chloride from the pharmacy and dilute before administration

A Bumetanide is a loop diuretic, which contributes to potassium loss and should be questioned. The Joint Commission has mandated that all concentrated electrolytes be mixed by a pharmacist and that vials of KCl should not be available in patient care areas. A large vein with high blood flow should be accessed to avoid phlebitis; it is recommended that the hand be avoided. Oral potassium supplements should be given with or following a snack or meal to avoid nausea.

The nurse is teaching a patient with congestive heart failure about monitoring fluid volume in the home setting. What are the best indicators of fluid losses or gains? A. Monitoring daily weights B. Observing ankles for swelling C. Monitoring daily intake and output D. Observing for pulse elevations with activity

A Changes in daily body weight are the best indicators of fluid losses or gains. Swelling of the ankles may occur due to a dependent position or standing for extended periods of time and not necessarily due to alterations in fluid volume. Monitoring intake and output may be useful but it is not as accurate as daily weights. The pulse normally rises with activities and decreases with rest. As such, observing the pulse with activity is not a good indicator of fluid status.

A nurse is assigned to care for a group of clients. After reviewing the medical records, which client should the nurse determine is at risk for deficient fluid volume? a. a client with a colostomy b. a client with heart failure c. a client with decreased kidney function d. a client receiving frequent wound irrigations

A Colostomy can result in the loss of gastrointestinal (GI) fluids, which predisposes the client to dehydration and a variety of electrolyte disturbances. When the colostomy bag is put in place, it is possible that the fluids may not be absorbed from the stools. Heart failure and decreased kidney function increase the client's risk of fluid volume overload. Frequent wound irrigations would not increase the client's risk for a fluid volume deficit.

A patient has a potassium level of 9.0 mEq/L. Which nursing intervention is priority? a. Prepare the patient for dialysis and place on a cardiac monitor b. administer spironolactone c. place on potassium-restricted diet d. administer a laxative

A Dialysis is used to help failing kidneys to be able to filter out waste, salt, and even extra water to prevent them from building up in the body, which will also help to keep the potassium levels down. Cardiac monitoring is required to monitor the heart activity related to the high potassium level. High potassium levels can increase the risk of ventricular and supraventricular arrhythmia, and if left untreated, can lead to sudden cardiac death.

The nurse is caring for a client with fluid volume excess. Which assessment finding will the nurse expect? a. distension of neck veins b. flat neck veins c. increased H&H values d. decreased central venous pressure readings

A In fluid volume excess, neck vein distention will occur. Flat neck veins indicate a fluid volume deficit. Increased hemoglobin and hematocrit values indicates hemoconcentration as an indication of fluid volume deficit. A low central venous pressure measurement occurs in fluid volume deficit.

Which assessment finding is likely in a patient reporting diarrhea and vomiting with an elevated body temperature of 103° F for 2 days who is brought to the emergency department? A. Urine output of 350 mL/day B. Oral mucosal membranes are dry C. Respiratory rate of 22 breaths per minute D. Neck veins distended when the patient is in supine positio

A Normal urine output should be not less than 500 mL/day. A urine output of 350 mL/day indicates dehydration. Dry oral mucus membranes can be a manifestation of dehydration, but can be misleading because some drugs also cause dry mouth as a side effect. A respiratory rate of 22 breaths per minute indicates an increased respiratory rate, which may be due to many other reasons. Neck veins are generally flat in patients with dehydration.

Which age-related change of the kidney increases an older adult patient's risk for dehydration? A. Increased water loss B. Increased thirst reflex C. Increased concentrating capacity D. Increased glomerular filtration rate

A Older adult patients are at an increased risk for dehydration caused by increased water loss. The older adult patient has a decreased, not an increased, thirst reflex which places them at risk for dehydration. Decreased, not increased, concentrating capacity and glomerular filtration rate also place the older adult patient at risk for dehydration.

The nurse is caring for a client with acute renal failure. For which electrolyte will the nurse monitor for an imbalance? a. Potassium b. Magnesium c. Phosphorus d. calcium

A With acute renal failure, the kidneys are unable to continuously remove excess potassium, which can lead to hyperkalemia. Acute renal failure is less likely to affect the magnesium, phosphorus, and calcium levels.

A nurse is caring for a client who has the following laboratory results: potassium 3.4 mEq/L, magnesium 1.8 mEq/L, calcium 8.5 mEq/L, sodium 144 mEq/L. Which assessment should the nurse complete first? a. Depth of respirations b. Bowel sounds c. Grip strength d. Electrocardiography

A A client with a low serum potassium level may exhibit hypoactive bowel sounds, cardiac dysrhythmias, and muscle weakness resulting in shallow respirations and decreased handgrips. The nurse should assess the clients respiratory status first to ensure respirations are sufficient. The respiratory assessment should include rate and depth of respirations, respiratory effort, and oxygen saturation. The other assessments are important but are secondary to the clients respiratory status.

A nurse is assessing clients on a medical-surgical unit. Which client is at risk for hypokalemia? a. Client with pancreatitis who has continuous nasogastric suctioning b. Client who is prescribed an angiotensin-converting enzyme (ACE) inhibitor c. Client in a motor vehicle crash who is receiving 6 units of packed red blood cells d. Client with uncontrolled diabetes and a serum pH level of 7.33

A A client with continuous nasogastric suctioning would be at risk for actual potassium loss leading to hypokalemia. The other clients are at risk for potassium excess or hyperkalemia.

A nurse assesses a client who is prescribed a medication that inhibits angiotensin I from converting into angiotensin II (angiotensin-converting enzyme [ACE] inhibitor). For which expected therapeutic effect should the nurse assess? a. Blood pressure decrease from 180/72 mm Hg to 144/50 mm Hg b. Daily weight increase from 55 kg to 57 kg c. Heart rate decrease from 100 beats/min to 82 beats/min d. Respiratory rate increase from 12 breaths/min to 15 breaths/min

A ACE inhibitors will disrupt the reninangiotensin II pathway and prevent the kidneys from reabsorbing water and sodium. The kidneys will excrete more water and sodium, decreasing the clients blood pressure.

A nurse is assessing clients for fluid and electrolyte imbalances. Which client should the nurse assess first for potential hyponatremia? a. A 34-year-old on NPO status who is receiving intravenous D5W b. A 50-year-old with an infection who is prescribed a sulfonamide antibiotic c. A 67-year-old who is experiencing pain and is prescribed ibuprofen (Motrin) d. A 73-year-old with tachycardia who is receiving digoxin (Lanoxin)

A Dextrose 5% in water (D5W) contains no electrolytes. Because the client is not taking any food or fluids by mouth (NPO), normal sodium excretion can lead to hyponatremia. The sulfonamide antibiotic, ibuprofen, and digoxin will not put a client at risk for hyponatremia.

A nurse is assessing a client with hypokalemia, and notes that the clients handgrip strength has diminished since the previous assessment 1 hour ago. Which action should the nurse take first? a. Assess the clients respiratory rate, rhythm, and depth. b. Measure the clients pulse and blood pressure. c. Document findings and monitor the client. d. Call the health care provider.

A In a client with hypokalemia, progressive skeletal muscle weakness is associated with increasing severity of hypokalemia. The most life-threatening complication of hypokalemia is respiratory insufficiency. It is imperative for the nurse to perform a respiratory assessment first to make sure that the client is not in immediate jeopardy. Cardiac dysrhythmias are also associated with hypokalemia. The clients pulse and blood pressure should be assessed after assessing respiratory status. Next, the nurse would call the health care provider to obtain orders for potassium replacement. Documenting findings and continuing to monitor the client should occur during and after potassium replacement therapy.

The patient with fluid overload has been taking a diuretic for the past 2 days and now experiences these changes. Which changes indicate to the nurse that the diuretic resulted in overdiuresis? Select all that apply. A. Reports of fatigue B. Weight loss of 9 pounds C. Heart rate increase from 70 to 96 beats/min D. Heart rate decrease from 80 to 72 beats/min E. Respiratory rate decrease from 20 to 16 breaths/min F. Reports of light-headedness when first standing from a lying position

A, B, C, F Diuretic drugs cause water loss and are often prescribed for edema. One liter of water weighs 2.2 pounds. Weight loss is expected when the patient gets rid of excess water. If the prescribed diuretic is overly effective, too much water may be excreted by the kidneys and signs of dehydration from overdiuresis may occur. A weight loss of 9 pounds is approximately 4 liters of fluid. Light-headedness when changing positions may suggest orthostatic hypotension. Other signs of too much fluid loss include an increase in heart rate and fatigue. The decrease in heart rate and respiratory rate would suggest the diuretic was effective.

A 92-year-old woman is admitted from a long-term care facility for treatment of dehydration. The provider has ordered fall precautions. Which interventions does the nurse implement as part of fall precautions? Select all that apply. A. Activate the bed alarm. B. Orient the patient frequently. C. Assess for orthostatic hypotension. D. Loosely apply upper extremity wrist restraints. E. Maintain a calm, dim room to reduce confusion. F. Place the bed in the lowest position with brakes locked.

A, B, C, F Multiple interventions are implemented to prevent falls, especially in older patients with dehydration. Assessing for orthostatic hypotension, orienting the patient frequently, placing the bed in the lowest position with the brakes locked, and activating the bed alarm should all be implemented to reduce the patient's risk of falling. In addition, frequent toileting and assistance to the bathroom may be indicated for this older patient. Restraints are never appropriate. A dimly lit room may increase the risk of falls.

A 77-year-old woman is brought to the emergency department by her family after she has had diarrhea for 3 days. The family tells the nurse that she has not been eating or drinking well, but that she has been taking her diuretics for congestive heart failure. She is receiving lactated Ringer's solution intravenously for rehydration. What clinical manifestations does the nurse monitor during rehydration of the patient? Select all that apply. A. Urinary output B. Blood pressure C. Blood serum glucose D. Pulse rate and quality E. Urine specific gravity levels

A, B, D, E The two most important areas to monitor during rehydration are pulse rate and quality and urine output; however, decreasing specific gravity of urine is also an indication of rehydration. Blood pressure is also important to monitor during rehydration. Blood glucose changes do not have a direct relation to a patient's rehydration status.

Which parameters should be monitored to assess a patient's response to fluid replacement therapy? Select all that apply. A. Monitoring pulse quality B. Monitoring urine output C. Monitoring serum bilirubin D. Monitoring white blood cell count E. Monitoring weight changes every 8 hours

A, B, E The patient who is receiving fluid replacement therapy should be monitored for adequate rehydration. Monitoring the pulse quality helps in assessing the patient's response to fluid therapy. Urine output should be monitored to assess the response of the patient to fluid therapy; urine output below 500 mL/day indicates dehydration. Weight changes are to be documented every 8 hours in order to prevent fluid overload. Alteration in serum bilirubin levels is not an indication of response to fluid therapy. Alteration of white blood cell count is not an indication of response to the fluid therapy.

An older adult patient with a history of renal failure is brought in to the emergency department with sudden onset of acute confusion, worsening muscle weakness in the extremities, abdominal cramps, and a weak, rapid, and thready pulse. What are the immediate nursing interventions to stabilize the patient? Select all that apply. A. Administering diuretics B. Administering 0.9% saline C Administering anti-psychotics D. Administering calcium channel blockers E. Administering 5% dextrose in 0.45% sodium chloride

A, B, E The patient's symptoms indicate hypernatremia. Administering diuretics that promote sodium loss and administering fluids such as 0.9% saline and 5% dextrose in 0.45% sodium chloride to restore the fluid balance are the immediate interventions needed to stabilize the patient. The patient's acute confusion is caused by high sodium levels, so anti-psychotic drugs should not be administered. Sodium decreases the heart contractibility by retarding the movement of calcium into the heart cells. Therefore calcium channel blockers should not be administered.

A nurse is assessing clients on a medical-surgical unit. Which clients are at increased risk for hypophosphatemia? (Select all that apply.) a. A 36-year-old who is malnourished b. A 42-year-old with uncontrolled diabetes c. A 50-year-old with hyperparathyroidism d. A 58-year-old with chronic renal failure e. A 76-year-old who is prescribed antacids

A, B, E Clients at risk for hypophosphatemia include those who are malnourished, those with uncontrolled diabetes mellitus, and those who use aluminum hydroxidebased or magnesium-based antacids. Hyperparathyroidism and chronic renal failure are common causes of hyperphosphatemia.

A nurse assesses a client who is admitted for treatment of fluid overload. Which manifestations should the nurse expect to find? (Select all that apply.) a. Increased pulse rate b. Distended neck veins c. Decreased blood pressure d. Warm and pink skin e. Skeletal muscle weakness

A, B, E Manifestations of fluid overload include increased pulse rate, distended neck veins, increased blood pressure, pale and cool skin, and skeletal muscle weakness.

A nurse is caring for clients with electrolyte imbalances on a medical-surgical unit. Which clinical manifestations are correctly paired with the contributing electrolyte imbalance? (Select all that apply.) a. Hypokalemia Flaccid paralysis with respiratory depression b. Hyperphosphatemia Paresthesia with sensations of tingling and numbness c. Hyponatremia Decreased level of consciousness d. Hypercalcemia Positive Trousseaus and Chvosteks signs e. Hypomagnesemia Bradycardia, peripheral vasodilation, and hypotension

A, C Flaccid paralysis with respiratory depression is associated with hypokalemia. Decreased level of consciousness is associated with hyponatremia. Paresthesia with sensations of tingling and numbness is associated with hypophosphatemia or hypercalcemia. Positive Trousseaus and Chvosteks signs are associated with hypocalcemia or hyperphosphatemia. Bradycardia, peripheral vasodilation, and hypotension are associated with hypermagnesemia.

Which mucous membrane assessments can the nurse use to evaluate a patient for dehydration? Select all that apply. A. Color B. Pulse C. Turgor D. Moisture E. Temperature

A, C, D When assessing the mucous membranes to evaluate a patient for dehydration, the nurse observes the color, moisture, and turgor of the mucous membranes. Although pulse and temperature can also indicate fluid status, these parameters are not directly related to assessment of mucous membranes. Pulse is a cardiovascular assessment and temperature is a central physiological assessment.

The nurse admits a patient with dehydration. Which electrolyte imbalances does the nurse anticipate based on this diagnosis? Select all that apply. A. Hyperkalemia B. Hypocalcemia C. Hypochloremia D. Hypernatremia E. Hypermagnesemia

A, D A patient who is admitted with dehydration will have hyperkalemia and hypernatremia. Hypocalcemia, hypochloremia, and hypermagnesemia are not expected electrolyte imbalances for a patient admitted with dehydration.

A nurse is assessing a client who has an electrolyte imbalance related to renal failure. For which potential complications of this electrolyte imbalance should the nurse assess? (Select all that apply.) a. Electrocardiogram changes b. Slow, shallow respirations c. Orthostatic hypotension d. Paralytic ileus e. Skeletal muscle weakness

A, D, E Electrolyte imbalances associated with acute renal failure include hyperkalemia and hyperphosphatemia. The nurse should assess for electrocardiogram changes, paralytic ileus caused by decrease bowel mobility, and skeletal muscle weakness in clients with hyperkalemia. The other choices are potential complications of hypokalemia.

Which age-related change to fluid balance does the nurse anticipate when conducting a neurologic assessment for an older adult patient? A. Decreased thirst reflex B. Increased urine output C. Increased muscle mass D. Decreased sweat production

A. An age-related neurologic change that impacts fluid balance is a decreased thirst reflex. Urine output, muscle mass, and sweat production may all impact fluid balance, but these are not age-related neurologic changes.

The primary health care provider prescribes intravenous administration of 100 mL of 20% glucose along with 20 units of insulin in a patient who is receiving furosemide therapy. What is the probable diagnosis of the patient? A. Hyperkalemia B. Hyperglycemia C. Hypernatremia D Hypercalcemia

A. Hyperkalemia is a condition where serum potassium levels are high. Potassium movement into the cells is enhanced by insulin. Intravenous administration of 100 mL 10% to 20% glucose with 10 to 20 units of regular insulin helps decrease serum potassium levels. Insulin increases the activity of sodium-potassium pumps, which decreases serum potassium levels temporarily by moving potassium from the extracellular fluid to the cells. This therapy is prescribed as an add-on therapy along with diuretics in a hyperkalemic patient. Conditions such as hyperglycemia, hypernatremia, and hypercalcemia cannot be managed with this insulin and glucose therapy.

A patient reports painful muscle spasms in the lower legs at rest, a tingling sensation in the hands and lips, and abdominal cramping and diarrhea. The nurse reviews the patient's laboratory results for the presence of which disorder? A. Hypocalcemia B. Hypernatremia C. Hypermagnesemia D. Hypophosphatemia

A. The primary symptoms of hypocalcemia are neuromuscular changes, specifically painful muscle cramps, and paresthesias that may spread to the face, progressing to tetany. Abdominal cramping and diarrhea may also occur. Muscle spasms in lower legs at rest, tingling sensation in the hands and lips, and abdominal cramping and diarrhea are not primary characteristics of hypernatremia, hypermagnesemia, or hypophosphatemia.

A patient has a potassium level of 2.0. What would you expect to be ordered for this patient?* A. Potassium 30 meq IV push B. Infusion of Potassium intravenously C. An oral supplement of potassium D. Intramuscular injection of Potassium

B

A patient has a sodium level of 123 and presents with confusion. The doctor diagnoses the patient with Syndrome of Inappropriate Antidiuretic Hormone Secretion (SIADH). Which type of hyponatremia is this? A. Hypovolemic B. Euvolemic C. Hypervolemic D. Antivolemic

B

The nurse is providing care for a patient who has hyperkalemia. Which disease process documented in the patient's medical record is the cause for this electrolyte imbalance? A. Cirrhosis B. Renal failure C. Diabetes mellitus D. Hyperaldosteronism

B Kidney disease, such as renal failure, causes a decrease in urine output, causing the patient to retain potassium and develop hyperkalemia. Cirrhosis is associated with hyponatremia, not hyperkalemia. Diabetes mellitus and hyperaldosteronism both cause an increase in urine output, which causes hypokalemia, not hyperkalemia.

The nurse is preparing a teaching plan for a client with low magnesium levels. Which foods should the nurse include as high sources of magnesium? a. butter or cheese b. spinach or butter beans c. grilled chicken sandwich or grilled lamb sandwich d. boiled macaroni or white rice

B Magnesium-rich foods would include green vegetables and beans, as well as bananas, nuts, and oranges. Foods like butter or cheese, grilled chicken or grilled lamb sandwiches, and boiled macaroni or white rice are not high in magnesium, but are typically high in sodium.

A patient with a sodium level of 112 is taking Lithium. Which of the following is a nursing priority? A. Hold further doses of Lithium B. Monitor Lithium drug level due to risk of toxicity C. Monitor potassium level due to increased risk of toxicity D. No priority is need. 112 is a normal sodium level

B Monitor Lithium drug level due to risk of toxicity

A nurse is caring for a client who exhibits dehydration-induced confusion. Which intervention should the nurse implement first? a. Measure intake and output every 4 hours. b. Apply oxygen by mask or nasal cannula. c. Increase the IV flow rate to 250 mL/hr. d. Place the client in a high-Fowlers position.

B Dehydration most frequently leads to poor cerebral perfusion and cerebral hypoxia, causing confusion. Applying oxygen can reduce confusion, even if perfusion is still less than optimal. Increasing the IV flow rate would increase perfusion. However, depending on the degree of dehydration, rehydrating the client too rapidly with IV fluids can lead to cerebral edema. Measuring intake and output and placing the client in a high-Fowlers position will not address the clients problem.

A 70-year-old woman is admitted to the hospital with heart failure, shortness of breath (SOB), and 3+ pitting edema in her lower extremities. Her medications are furosemide, digoxin, and an angiotensin-converting enzyme (ACE) inhibitor. She states that she stopped taking her furosemide because she did not think that it was helping her heart failure. Her health care provider orders furosemide 5 mg IV push. Ten hours after receiving the furosemide, the patient's potassium (K+) level is 2.5 mEq/L. Knowing all of the patient's medications, what problem does the nurse anticipate in this patient? A. Signs and symptoms of hypernatremia B. Clinical manifestations of digoxin toxicity C. Increased heart rate and blood pressure (BP) D. Increased signs of congestive heart failure (CHF

B Hypokalemia increases the sensitivity of cardiac muscle to digoxin and may result in digoxin toxicity, even when the digoxin level is within the therapeutic range. Heart rate and BP would be more likely to decrease with the medications that the patient is receiving coupled with her low potassium level. Use of a diuretic tends to decrease the signs of CHF. High serum sodium levels would not be expected in this scenario.

A nurse is assessing clients on a medical-surgical unit. Which adult client should the nurse identify as being at greatest risk for insensible water loss? a. Client taking furosemide (Lasix) b. Anxious client who has tachypnea c. Client who is on fluid restrictions d. Client who is constipated with abdominal pain

B Insensible water loss is water loss through the skin, lungs, and stool. Clients at risk for insensible water loss include those being mechanically ventilated, those with rapid respirations, and those undergoing continuous GI suctioning. Clients who have thyroid crisis, trauma, burns, states of extreme stress, and fever are also at increased risk. The client taking furosemide will have increased fluid loss, but not insensible water loss. The other two clients on a fluid restriction and with constipation are not at risk for fluid loss.

Which assessment findings would indicate decreased tissue perfusion stimulating the secretion of renin from the kidney tubular system? Select all that apply. A. High urine output B. Low blood volume C. Low blood oxygen D. High blood sodium E. High blood pressur

B, C Low blood volume and low blood oxygen are two conditions that signal decreased tissue perfusion, which stimulates the secretion of renin from the kidney tubular system. High urine output, high sodium, and high blood pressure are not conditions that indicate decreased tissue perfusion, which would stimulate the secretion of renin from the kidney tubular system.

patient is admitted to the intensive care unit with a diagnosis of syndrome of inappropriate antidiuretic hormone secretion (SIADH). Which treatments does the nurse anticipate for this patient? Select all that apply. A. Digitalis (Lanoxin) B. Furosemide (Lasix) C. Tolvaptan (Samsca) D. Conivaptan (Vaprisol) E. Vasopressin (Pitressin) F. Norepinephrine (Levophed

B, C, D The nurse anticipates the health care provider will prescribe furosemide, tolvaptan, and conivaptan to manage SIADH. Furosemide is a loop-diuretic that helps rid the body of excess fluid. Tolvaptan is used to manage hyponatremia with SIADH. Conivaptan acts as an antidiuretic hormone (ADH) inhibitor to help resolve fluid volume overload. Digitalis is a cardiac glycoside that slows and strengthens myocardial contraction, but it has no effect on fluid volume. Vasopressin is contraindicated with SIADH since it is the synthetic form of ADH. Norepinephrine is a powerful peripheral vasoconstrictor used to increase blood pressure.

Which will the nurse include when documenting a patient's fluid intake for the shift? Select all that apply. A. Emesis B. Enemas C. Oral fluids D. Solid foods E. Irrigation fluids

B, C, E Intake that is considered measurable when documenting a patient's fluid intake for the shift include enemas, oral fluids, and irrigation fluids. Emesis is a measurable output. Solid foods are considered intake but these cannot be measured.

A patient reports swelling of the right foot and ankle. Upon assessing the patient, the health care provider confirms it as pitting edema and prescribes diuretic therapy. Which nursing interventions are necessary for this patient? Select all that apply. A. Monitoring the respiratory rate B. Monitoring the urine output of the patient C. Assessing the sodium and potassium values D. Checking the urine for correct specific gravity E. Monitoring the electrocardiogram patterns (ECG

B, C, E Patients with fluid overload often have pitting edema, and diuretic therapy focuses on removing the excess fluid. The nursing interventions would be monitoring the patient's response to drug therapy, especially increased urine output and weight loss. Diuretic therapy is associated with electrolyte imbalance; therefore sodium and potassium levels need to be monitored. Severe electrolyte disturbances may result in arrhythmias. Therefore changes in the electrocardiogram (ECG) should be monitored. Diuretic therapy does not cause respiratory depression or changes in respiratory rate, so the respiratory rate does not need to be monitored. Checking the urine specific gravity is beneficial in patients to detect the fluid overload. However, it is not useful in patients on diuretic therapy.

Which statement by the nurse is most appropriate when teaching a patient about maintaining fluid balance? A. "If you increase your intake of salt, you are at a greater risk for dehydration." B "Your intake and output has the most influence on your body's fluid balance." C. "If you decrease your intake of potassium, you are at a greater risk for dehydration." D. "Your prescribed medications have the largest impact on fluid balance within the body."

B. Body fluids are constantly filtered and replaced as fluid balance is maintained through intake and output. An increased sodium intake will increase the risk for fluid overload, not dehydration. An increased rather than decreased intake of potassium places a patient at risk for dehydration. While certain medications, such as diuretics, can impact the body's fluid balance, it is the actual intake and output that has the largest impact on fluid balance.

A patient who is receiving intravenous fluid replacement therapy has a bounding pulse, difficulty in breathing, and distended neck veins when sitting in the upright position. What is the most appropriate nursing intervention for the patient? A. Monitoring for weight changes B. Administering diuretic drug therapy C. Increasing fluid replacement therapy D. Continuing fluid replacement therapy

B. Bounding pulse, difficulty in breathing, and distention of neck veins in the upright position indicates fluid overload. Diuretics help increase the excretion of fluids from the body, thereby maintaining the fluid balance and avoiding the risk of fluid overload. Weight change is the best indicator of fluid losses or gains but is not an appropriate intervention. Increasing, or even continuing, the fluid replacement therapy can potentiate further fluid overload and may worsen the condition and lead to pulmonary edema or even death.

A patient with a history of hypertension asks the nurse what dietary changes are necessary to make in order to control the blood pressure. What does the nurse include in the instruction? A. Reduce the intake of iron B. Reduce the intake of calcium C Reduce the intake of sodium D Reduced the intake of phosphorous

C High sodium intake raises the serum sodium level, which causes more water to be retained. This in turn increases the blood volume and raises the blood pressure. Hence, patients who have hypertension are often asked to limit their intake of sodium. Intake of iron, phosphorus, or calcium does not cause water retention in the blood and therefore does not affect the blood pressure.

Which condition could be evident in laboratory reports of a hypervolemic patient? A. Hemostasis B. Homeostasis C. Hemodilution D. Hemoconcentration

C Hypervolemia or fluid overload is characterized by decreased hemoglobin, hematocrit, and serum protein levels due to excessive water in the vascular space. This condition is called hemodilution. Hemostasis and homeostasis are not associated with hypervolemia or fluid overload. Hemoconcentration is the condition associated with hypovolemia or dehydration.

A patient admitted with fluid overload from heart failure is treated with diuretic therapy. What laboratory test demonstrates effectiveness of the diuretic therapy? A. Serum osmolality B. Serum creatinine C. Natriuretic peptide D. Serum magnesium

C Natriuretic peptides (atrial and brain natriuretic peptides) are hormones secreted by cells that line the atria and ventricles of the heart in response to changes in blood volume and pressure. Serum osmolality may be monitored if the patient is over- or underdiuresed. Serum magnesium and creatinine would not be indicated as a measure of diuretic therapy.

A client recovering from a thyroidectomy reports a new onset of numbness and tingling around the mouth. Which finding indicates the client's serum calcium level is low? a. bone pain b. depressed DTR c. positive Chvostek's sign d. nausea

C Signs of hypocalcemia include numbness and tingling, typically around the mouth, tongue, fingers and feet, bone pain, nausea, and depressed deep tendon reflexes. If the facial muscles contract when the facial nerve in front of the ear is tapped, this is considered a positive Chvostek's sign, a positive indication of hypercalcemia.

A nurse teaches clients at a community center about risks for dehydration. Which client is at greatest risk for dehydration? a. A 36-year-old who is prescribed long-term steroid therapy b. A 55-year-old receiving hypertonic intravenous fluids c. A 76-year-old who is cognitively impaired d. An 83-year-old with congestive heart failure

C Older adults, because they have less total body water than younger adults, are at greater risk for development of dehydration. Anyone who is cognitively impaired and cannot obtain fluids independently or cannot make his or her need for fluids known is at high risk for dehydration.

The RN is assessing a 70-year-old patient admitted to the unit with severe dehydration. Which finding requires immediate intervention by the nurse? A. Deep furrows on the surface of the tongue B. Urine output of 950 mL for the past 24 hours C. Patient behavior that changes from anxious and restless to lethargic and confused D. Poor skin turgor with tenting remaining for 2 minutes after the skin is pinched

C. The patient's change in level of consciousness from anxious and restless to lethargic and confused suggests poor cerebral blood flow, or shrinkage or swelling of brain cells caused by fluid shifts within the brain cells. These changes indicate a need for immediate intervention to prevent further damage to cerebral function. Deep furrows on the surface of the tongue, poor skin turgor, and low urine output are all caused by the fluid volume deficit but do not indicate complications of dehydration that are immediately life-threatening.

Which patient below is at risk for experiencing Hypovolemic Hyponatremia? A. Patient with congestive heart failure B. Patient with cirrhosis of the liver C. Patient on IV saline at 250 cc/hr D. Patient with nasogastric tube suction experiencing diarrhea

D

A client has a serum sodium level of 110 mEq/L. Which nursing intervention should be implemented? a. encourage fluids orally b. administer 10% saline IVPB c. administer antidiuretic hormone intranasally d. place on seizure precautions

D Clients with low sodium levels are at risk for seizures as a complication. The lower the sodium level, the greater the risk for seizures. When a client has a low sodium level, fluids are not given as this may further decrease the level. An antidiuretic hormone could cause water retention and hypertonic saline solutions should only be used very cautiously, because if the sodium levels increase too rapidly, a massive fluid shift can occur in the body, resulting in neurological damage and heart failure.

Which person attending an all-day outdoor festival on a hot August day is at greatest risk for dehydration? A. 32-year-old man consuming alcoholic beverages B. 28-year-old woman with type 1 diabetes mellitus C. 68-year-old man who frequently wipes sweat from his forehead D. 72-year-old woman appearing to be at least 20 pounds overweigh

D The thirst mechanism is less sensitive in older adults, making them more at risk for dehydration. Women of any age have less total body water than men of similar sizes and ages, because men have more muscle mass than women and women have more body fat. (Muscle cells contain mostly water, and fat cells have little water.) In addition, the 72-year-old woman is overweight, with an increased percentage of body fat compared to lean body mass, especially skeletal muscle. An obese person has less total water than a lean person of the same weight because fat cells contain almost no water. Although a 32-year-old man consuming alcohol, a 28-year-old with type 1 diabetes, and a 68-year-old who frequently sweats are at risk for dehydration, they are not as high-risk as the obese older adult.

A patient presents with dehydration. Which parameter should be considered when deciding isotonic fluid replacement therapy for the patient? A. Urine output of 550 mL per day B. Heart rate of 100 beats per minute C. Respiratory rate of 18 breaths per minute D Body temperature of 40°C for more than 8 hrs

D A body temperature of 40°C is abnormally high, which causes dehydration in patients. To replace the fluid loss caused by dehydration, the nurse administers isotonic fluid replacement therapy. A urine output of 550 mL per day is the optimal urine output required to eliminate toxic substances. A heart rate of 100 beats per minute is the normal heart rate of an adult and does not indicate fluid replacement therapy. A respiratory rate of 18 breaths per minute is the normal respiratory rate in adults and does not indicate the need for replacement fluid.

A nurse is evaluating a client who is being treated for dehydration. Which assessment result should the nurse correlate with a therapeutic response to the treatment plan? a. Increased respiratory rate from 12 breaths/min to 22 breaths/min b. Decreased skin turgor on the clients posterior hand and forehead c. Increased urine specific gravity from 1.012 to 1.030 g/mL d. Decreased orthostatic light-headedness and dizziness

D The focus of management for clients with dehydration is to increase fluid volumes to normal. When fluid volumes return to normal, clients should perfuse the brain more effectively, therefore improving confusion and decreasing orthostatic light-headedness or dizziness. Increased respiratory rate, decreased skin turgor, and increased specific gravity are all manifestations of dehydration.

A patient with a sodium level of 178 is ordered to be started on 0.45% Saline. What is the most IMPORTANT nursing intervention for this patient? A. Maintain patent IV B. Give rapidly to ensure fluids levels are shifted properly C. Clarify doctor's order because 0.45% saline is contraindicated in hypernatremia D. Give slowly and watch for signs and symptoms of cerebral edema

D: Give slowly and watch for signs and symptoms of cerebral edema. The most important intervention is to give slowly and watch for S&S of cerebral edema because a hypotonic solution can cause rapid swelling of the cell. Maintain a patent IV is correct but not the most important option in this scenario.

Which nursing action is recommended when providing care to a patient with hypokalemia? a. Question the continued administration of bumetanide. b. Administer prescribed oral potassium chloride before a meal or snack. c. Establish a peripheral IV, preferably in the hand, for administering IV potassium chloride. d. Obtain the prescribed vial of IV potassium chloride from the pharmacy and dilute before administration.

a Bumetanide is a loop diuretic, which contributes to potassium loss and should be questioned. The Joint Commission has mandated that all concentrated electrolytes be mixed by a pharmacist and that vials of KCl should not be available in patient care areas. A large vein with high blood flow should be accessed to avoid phlebitis; it is recommended that the hand be avoided. Oral potassium supplements should be given with or following a snack or meal to avoid nausea.

which electrolyte imbalance does the nurse anticipate in association with a serum magnesium reading of 1.1 mEq/L? a. Calcium 7.8 mg/dL b. Sodium 149 mEq/L c. Potassium 5.7 mEq/L d. Phosphorus 2.6 mg/dL

a Hypocalcemia often occurs with hypomagnesemia. A calcium level of 7.8 mg/dL is low. A sodium level of 149 mEq/L is slightly elevated, but not related to the low magnesium level. A phosphorus level of 2.6 mg/dL is slightly low, but not related to hypomagnesemia. A potassium level of 5.7 is elevated, but not related to low magnesium levels.

The nurse finds that a patient has dry skin, a heart rate of 115 beats per minute, a respiratory rate of 28 breaths per minute, and weight loss of 1 pound in 1 day. What does the nurse infer about the patient's condition? a. The patient is suffering from dehydration. b. The patient is suffering from hyperkalemia. c. The patient is suffering from fluid overload. d. The patient is suffering from hyponatremia.

a The best indicator of fluid loss or fluid overload is daily weight. Decrease in weight may occur due to fluid losses, which may affect the cardiovascular system. Therefore heart rate increases in an attempt to maintain blood pressure when the blood volume decreases. Dehydrationdecreases the blood volume, which may result in increased respiratory rate. Dry skin also indicates dehydration. Hyperkalemia causes cardiovascular effects such as bradycardia, hypotension, and arrhythmias. Fluid overload causes edema and increase in weight. Hyponatremia causes cerebral changes such as cerebral edema and increased intracranial pressure.

The RN is caring for a patient admitted with dehydration who requires a blood transfusion. Which nursing action does the RN delegate to unlicensed assistive personnel (UAP)? a. Obtaining baseline vital signs before blood administration b. Inserting a small-gauge needle for intravenous (IV) access c. Evaluating a headache that develops during the transfusion d. Explaining to the patient the purpose of the blood transfusion

a UAP education includes assessment of vital signs, so obtaining vital signs is within their scope of practice. IV starts, evaluating patient symptoms, and explaining the purpose of a blood transfusion require broader education and scope of practice and should be done by licensed staff members.

When treating a patient for hyponatremia, which type of drug must be altered to decrease sodium loss? a. Diuretics d. Biphosphates c. Corticosteroids d. Beta-adrenergic agonists

a When treating a patient with hyponatremia, if the patient is already taking diuretics, his or her dosage must be adjusted because diuretics increase sodium loss. Biphosphates are used to prevent hypercalcemia. Corticosteroids can cause hypernatremia. Beta-adrenergic agonists can cause hypokalemia.

Which nursing interventions are consistent with safe administration of intravenous (IV) potassium to a patient with hypokalemia? Select all that apply. a. Evaluate the heart rate and regularity. b. Establish and evaluate the patency of a large vein. c. Obtain an IV controller device (pump). d. Plan to assess the respiratory rate and oxygen saturation every hour. e. Prepare to administer potassium IV push to reduce the risk of infiltration. f. Encourage the patient to ambulate independently to relieve muscle cramps.

a, b, c, d Pulse irregularities (rapid to slow and irregular) may occur with changes in serum potassium levels and should be evaluated. To safely administer IV potassium, the nurse would ensure that the patient has IV access in a large vein if possible, obtain an IV pump to regulate the infusion rate at no greater than 10 mEq of potassium per hour, and evaluate the patient's respiratory status hourly during and immediately following infusion (as respiratory insufficiency is the major cause of death). Potassium should never be given IV push, as it will result in cardiac arrest. Patients with hypokalemia have skeletal muscle weakness, so fall precautions should be implemented, and the patient should have assistance with ambulation.

The patient with fluid overload has been taking a diuretic for the past 2 days and now experiences these changes. Which changes indicate to the nurse that the diuretic resulted in overdiuresis? Select all that apply. a. Reports of fatigue b. Weight loss of 9 pounds c. Heart rate increase from 70 to 96 beats/min d. Heart rate decrease from 80 to 72 beats/min e. Respiratory rate decrease from 20 to 16 breaths/min f. Reports of light-headedness when first standing from a lying position

a, b, c, f Diuretic drugs cause water loss and are often prescribed for edema. One liter of water weighs 2.2 pounds. Weight loss is expected when the patient gets rid of excess water. If the prescribed diuretic is overly effective, too much water may be excreted by the kidneys and signs of dehydration from overdiuresis may occur. A weight loss of 9 pounds is approximately 4 liters of fluid. Light-headedness when changing positions may suggest orthostatic hypotension. Other signs of too much fluid loss include an increase in heart rate and fatigue. The decrease in heart rate and respiratory rate would suggest the diuretic was effective.

Which findings would indicate the effectiveness of furosemide therapy prescribed for a patient diagnosed with fluid overload? Select all that apply. a. Decreased weight b. Increased pulse rate c. Increased urine output d. Increased blood pressure e. Increased neck vein distention

a, c Weight loss and increased urine output indicate fluid loss from the body, which indicates the effectiveness of drug therapy. Increased pulse rate is a manifestation of fluid overload, which shows the ineffectiveness of the therapy. Decreased blood pressure is a manifestation of dehydration. Increased neck vein distention is a clinical manifestation of fluid overload, which shows that the therapy is ineffective in correcting the fluid balance.

The nurse reviews a client's electrolyte laboratory report and notes that the potassium level is 2.5 mmol/L). Which patterns should the nurse watch for on the electrocardiogram (ECG) as a result of the laboratory value? Select all that apply. a. U waves b. Absent P waves c. Inverted T waves d.Depressed ST segment e.Widened QRS complex

a, c, d The normal serum potassium level is 3.5 to 5.0 mEq/L (3.5 to 5.0 mmol/L). A serum potassium level lower than 3.5 mEq/L (3.5 mmol/L) indicates hypokalemia. Potassium deficit is an electrolyte imbalance that can be potentially life-threatening. Electrocardiographic changes include shallow, flat, or inverted T waves; ST segment depression; and prominent U waves. Absent P waves are not a characteristic of hypokalemia but may be noted in a client with atrial fibrillation, junctional rhythms, or ventricular rhythms. A widened QRS complex may be noted in hyperkalemia and in hypermagnesemia.

Which mucous membrane assessments can the nurse use to evaluate a patient for dehydration? Select all that apply. a. Color b. Pulse c. Turgor d. Moisture e. Temperature

a, c, d When assessing the mucous membranes to evaluate a patient for dehydration, the nurse observes the color, moisture, and turgor of the mucous membranes. Although pulse and temperature can also indicate fluid status, these parameters are not directly related to assessment of mucous membranes. Pulse is a cardiovascular assessment and temperature is a central physiological assessment.

Which signs does the nurse expect to find when assessing a patient with fluid overload? Select all that apply. a. Weight gain b. Hypotension c. Crackles in the lungs d. Large amount of dilute urine e. Pitting edema in the ankles and feet

a, c, e Signs of fluid overload are hypertension, hemodilution, pulmonary crackles, peripheral edema, and decreased urinary output. Rapid weight gain is the most reliable indicator of fluid retention and overload.

A patient is being discharged from the hospital before completely resolving the problem of fluid overload. What instructions should the nurse give to the caretaker in order to avoid the recurrence of fluid overload? Select all that apply. a. Monitor the fluid intake. b. Monitor the protein intake. c. Monitor the sodium intake. d. Monitor the carbohydrate intake. e. Monitor the weight gain of the patient.

a, c, e The main causes of fluid overload are excess fluid and sodium intake. Weight gain is the best indicator of fluid retention and overload. So the intake of fluid and sodium should be monitored in a patient with continuing risk of fluid overload. Each pound of weight gain equates to approximately 500 mL of retained fluid. So monitoring the weight gain in the patient with continuing risk of fluid overload is mandatory. Monitoring the intake of proteins and carbohydrates in a patient does not have any significance in reducing the risk of fluid overload.

Hyponatremia most affects the cells of which body systems? Select all that apply. a. Cerebral b. Endocrine c. Respiratory d. Cardiovascular e. Neuromuscular

a, d, e The cells of the cerebral, cardiovascular, and neuromuscular systems are most affected by hyponatremia. The cells of the endocrine and respiratory systems are not as affected.

A patient has a serum magnesium level of 1.2 mEq/L. Which instruction by the nurse is appropriate? a. "Notify me if you have diarrhea." b. "I will be giving you an intramuscular injection of magnesium sulfate." c. "Your urine may become darker and more concentrated in appearance." d. "I will be administering a phosphorus supplement in addition to the magnesium supplement."

a. A serum magnesium level of 1.2 mEq/L represents mild hypomagnesemia (normal is 1.3-2.1 mEq/L) for which oral magnesium supplements are administered. Oral magnesium may lead to diarrhea; diarrhea contributes to magnesium loss. With severe hypomagnesemia, the IV route (instead of IM) is used because the IM route causes tissue damage and pain. Any medications containing phosphorus are contraindicated because they would contribute to associated hypocalcemia. Magnesium therapy does not affect the urine.

The nurse is assessing a client with a lactose intolerance disorder for a suspected diagnosis of hypocalcemia. Which clinical manifestation would the nurse expect to note in the client? a. Twitching b. Hypoactive bowel sounds c.Negative Trousseau's sign d.Hypoactive deep tendon reflexes

a. A client with lactose intolerance is at risk for developing hypocalcemia, because food products that contain calcium also contain lactose. The normal serum calcium level is 9 to 10.5 mg/dL (2.25 to 2.75 mmol/L). A serum calcium level lower than 9 mg/dL (2.25 mmol/L) indicates hypocalcemia. Signs of hypocalcemia include paresthesias followed by numbness, hyperactive deep tendon reflexes, and a positive Trousseau's or Chvostek's sign. Additional signs of hypocalcemia include increased neuromuscular excitability, muscle cramps, twitching, tetany, seizures, irritability, and anxiety. Gastrointestinal symptoms include increased gastric motility, hyperactive bowel sounds, abdominal cramping, and diarrhea

A patient has a serum magnesium level of 1.2 mEq/L. Which instruction by the nurse is appropriate? a. "Notify me if you have diarrhea." b. "I will be giving you an intramuscular injection of magnesium sulfate." c. "Your urine may become darker and more concentrated in appearance." d. "I will be administering a phosphorus supplement in addition to the magnesium supplement."

a. A serum magnesium level of 1.2 mEq/L represents mild hypomagnesemia (normal is 1.3-2.1 mEq/L) for which oral magnesium supplements are administered. Oral magnesium may lead to diarrhea; diarrhea contributes to magnesium loss. With severe hypomagnesemia, the IV route (instead of IM) is used because the IM route causes tissue damage and pain. Any medications containing phosphorus are contraindicated because they would contribute to associated hypocalcemia. Magnesium therapy does not affect the urine.

Which nursing action is recommended when providing care to a patient with hypokalemia? a. Question the continued administration of bumetanide. b. Administer prescribed oral potassium chloride before a meal or snack. c. Establish a peripheral IV, preferably in the hand, for administering IV potassium chloride. d. Obtain the prescribed vial of IV potassium chloride from the pharmacy and dilute before administration

a. Bumetanide is a loop diuretic, which contributes to potassium loss and should be questioned. The Joint Commission has mandated that all concentrated electrolytes be mixed by a pharmacist and that vials of KCl should not be available in patient care areas. A large vein with high blood flow should be accessed to avoid phlebitis; it is recommended that the hand be avoided. Oral potassium supplements should be given with or following a snack or meal to avoid nausea.

Laboratory results report a patient's serum potassium at 5.6 mEq/L. What does the nurse immediately assess in the patient? a. Heart rate b. Bowel sounds c. Feet for paresthesias d. Level of consciousness

a. Cardiovascular changes, specifically bradycardia; tall, peaked T waves; rhythm changes to complete heart block; asystole; and ventricular fibrillation are life-threatening consequences of elevated potassium. The provider or Rapid Response Team may need to be notified if changes in heart rate and rhythm are assessed. Paresthesias in the arms and feet and increased intestinal motility are lower-priority signs of elevated potassium. Level of consciousness would not be affected.

A patient develops fluid overload while in the intensive care unit. Which nursing intervention does the nurse perform first? a. Elevates the head of the bed b. Draws blood for laboratory tests c. Puts the patient in a side-lying position d. Places the extremities in a dependent position

a. Elevating the head of the bed will ease breathing for the patient, so it should be done first. Although drawing blood for laboratory tests may be indicated, the nurse should perform interventions that will help with physiologic changes caused by fluid overload first. Placing the extremities in a dependent position increases peripheral edema, and positioning the patient in a side-lying position increases the work of breathing.

Which electrolyte excess results in irritability and severe cellular dehydration? a. Sodium b. Calcium c. Phosphorus d. Magnesium

a. Hypernatremia occurs when serum sodium levels are very high; this condition causes excitable tissues to over-respond to stimuli. This leads to irritability and severe cellular dehydration. Hypercalcemia, an excess of calcium, decreases the sensitivity of excitable tissues to normal stimuli. Hyperphosphatemia (excess phosphorus) causes hypocalcemia. Hypermagnesemia (excess magnesium) causes hypotension, bradycardia, central nervous system changes, and neuromuscular changes.

The nurse is analyzing a patient's serum electrolyte results. Which lab result is of the highest concern? a. Chloride 88 mEq/L b. Sodium 143 mEq/L c. Potassium 4.8 mEq/L d. Magnesium 2.0 mEq/L

a. The chloride level of 88 mEq/L is low, so it is of the greatest concern to the nurse. The normal chloride range is 98-106 mEq/L. A decreased range is indicative of fluid loss that may be the result of dehydration, vomiting, or diarrhea. The sodium, magnesium, and potassium levels are within the normal ranges.

When administering 20 mEq potassium chloride intravenously (IV), which is the priority intervention? a. Administer at a rate of 10 mEq/hr. b. Monitor respiratory rate and depth. c. Monitor for pain or burning at the IV infusion site. d. Place the patient on a heart monitor during administration.

a. The maximum recommended infusion rate is 5 to 10 mEq/hr to avoid potentially lethal cardiac dysrhythmias. Monitoring for pain at the IV infusion site, assessing respiratory rate and depth, and placing the patient on a heart monitor are all appropriate options, but because a rapid rate of administration could have lethal effects, it has the greatest priority.

The RN is caring for a patient admitted with dehydration who requires a blood transfusion. Which nursing action does the RN delegate to unlicensed assistive personnel (UAP)? a. Obtaining baseline vital signs before blood administration b. Inserting a small-gauge needle for intravenous (IV) access c. Evaluating a headache that develops during the transfusion d. Explaining to the patient the purpose of the blood transfusion

a. UAP education includes assessment of vital signs, so obtaining vital signs is within their scope of practice. IV starts, evaluating patient symptoms, and explaining the purpose of a blood transfusion require broader education and scope of practice and should be done by licensed staff members.

an older adult patient is admitted with dehydration. Which nursing assessment data identify that the patient is at risk for falling? a. Dry oral mucous membranes b. Orthostatic blood pressure changes c. Serum potassium level of 4.0 mEq/L d. Pulse rate of 72 beats/min and bounding

b Blood pressure decreases when changing positions. The patient may not have sufficient blood flow to the brain, causing sensations of light-headedness and dizziness. This problem increases the risk for falling, especially in older adults. Assessment of oral mucous membranes and the pulse rate can detect symptoms of dehydration, but these are not the best ways to assess for a fall risk. Checking serum potassium does not assess for fall risk.

When assessing the laboratory results of a patient who has hypomagnesemia, for which additional electrolyte imbalance should the nurse monitor? a. Hyperkalemia b. Hypocalcemia c. Hypernatremia d. Hypophosphatemia

b Hypocalcemia often occurs with hypomagnesemia, so the nurse would monitor for signs and symptoms of low calcium levels. Hypomagnesemia may increase potassium secretion in certain circumstances, leading the health care provider to be aware that replacement of magnesium is crucial before attempting to replace potassium if the patient is deficient in both. Hypernatremia and hypophosphatemia are not related to hypomagnesemia.

The nurse is assessing a patient with hyponatremia. Which finding requires immediate action? a. Heightened acuity b. Muscular weakness c. Urine output of 35 mL/hr d. Diminished bowel sounds

b Muscle weakness in patients with hyponatremia requires immediate action. If muscle weakness is present, immediately check respiratory effectiveness because ventilation is dependent on adequate strength of the respiratory muscles. Excessive bowel sounds, not diminished bowel sounds, are expected in the patient with hyponatremia, as well as mild confusion, not heightened acuity. A urine output of 35 mL/hr is normal (minimally) and does not require immediate action.

A patient who recently experienced an anterior neck injury reports frequent and painful muscle spasms in the calf during sleep. Which condition does the nurse suspect in the patient? a. Hypokalemia b. Hypocalcemia c. Hyponatremia d. Hypophosphatemia

b Patients with a history of anterior neck injury are at a high risk for hypocalcemia. Frequent painful muscle spasms in the calf or foot during rest or sleep (charley horses) indicate hypocalcemia. Hypokalemia, hyponatremia, and hypophosphatemia do not cause painful calf muscle spasms.

An older adult patient who is prescribed diuretics and laxatives and has a history of diabetes mellitus is brought to the emergency room in an unconscious state. Which is the most likely reason for the patient's condition? a. Edema b. Dehydration c. Fluid overload d. Water intoxication

b Patients with diabetes mellitus may have polyuria, which causes loss of body water. Diuretics and laxatives also cause loss of body water, further potentiating dehydration, and may lead to unconsciousness in older adult patients. Edema is the retention of fluid in the body. Fluid overload is caused due to excessive fluid intake or impaired renal or cardiac function. Water intoxication may result due to excess intake of water without adequate intake of sodium.

The patient is a 69-year-old woman with uncontrolled diabetes, polyuria, and a blood pressure of 86/46 mm Hg. Which staff member is assigned to care for her? a. RN who usually works as a diabetic educator b. RN who has floated from the intensive care unit c. LPN/LVN who has floated from the hospital's long-term care unit d LPN/LVN who frequently administers medications to multiple patients

b The clinical manifestations suggest that the patient is experiencing hypovolemia and possible hypovolemic shock. The RN who floated from the intensive care unit will have extensive experience caring for patients with hypovolemia. The LPN/LVN who has floated from the long-term care unit or who frequently administers medications to multiple patients will not be as familiar with care for critically ill patients. The LPN/LVN is not qualified to care for a patient with these complications. Although the resource on diabetes is helpful, this RN will not be as familiar with care for critically ill patients.

What is the function of aldosterone in the body? a. It causes constriction of renal arterioles. b. It promotes resorption of water and sodium. c. It stimulates secretion of renin for the kidneys. d. It causes constriction of peripheral blood vessels.

b Aldosterone promotes reabsorption of sodium and water into the body, which helps in maintaining blood pressure. Angiotensin II causes constriction of renal arterioles, resulting in low urine output. Factors such as low blood pressure, low blood volume, low oxygen, and low sodium trigger secretion of renin. Angiotensin II causes constriction of peripheral blood vessels and helps in maintaining perfusion to vital organs.

A hypertensive patient was brought to the emergency department with a heart rate of 115 beats per minute and an abnormal electrocardiogram showing a shortened QT interval. The laboratory findings of the patient show a serum calcium level of 11 mg/dL. What nursing interventions would help stabilize the patient? Select all that apply. a. Administering thiazide diuretics b. Administering high ceiling or loop diuretics c. Administering 0.9% normal saline intravenously d. Administering nonsteroidal, anti-inflammatory agents e. Administering lactated Ringer's solution intravenously

b, c Hypercalcemia clinically manifests as serum calcium levels above 10.5 mg/dL. This condition can be managed by using loop diuretics such as furosemide, which promote the excretion of calcium. One cause of hypercalcemia is dehydration, which can be well managed by administering 0.9% normal saline intravenously. Thiazide diuretics do not promote the excretion of calcium and thus are not suitable treatments for hypercalcemia. Lactated Ringer's solution contains calcium; administering this solution does not help decrease the serum calcium levels. Administering nonsteroidal, anti-inflammatory agents prevents hypercalcemia by calcium resorption from the bone; however, it does not treat hypercalcemia.

Which assessment findings will indicate the need for continuation of prescribed fluid replacement therapy in a patient diagnosed with dehydration? Select all that apply. a. Hemoglobin of 13.5 g/dL b. Pulse pressure of 28 mm Hg c. Urine output of 400 mL per day d. Respiratory rate of 20 breaths per minute e. Neck veins distention when the patient is in a supine position

b, c Pulse pressure below 40 mm Hg and urine output below 500 mL are abnormal and require continuation of the fluid replacement therapy. Hemoglobin of 13.5 g/dL is within the normal range. Respiratory rate of 20 breaths per minute is also a normal finding. Neck veins distention is evidence of fluid overload.

Which patient is at increased risk for fluid and electrolyte imbalance? Select all that apply. a. 22-year-old pregnant woman in her third trimester b. 24-year-old male running athlete c. 45-year-old man on diuretics d. 47-year-old man traveling to Florida in summer e. 76-year-old bedridden woman

b, c, e A 24-year-old male running athlete is at risk for dehydration, therefore would put him at risk for fluid and electrolyte imbalances. A 45-year-old man on diuretics is at risk for fluid and electrolyte imbalances owing to the action(s) of the drugs. Many of the high-ceiling (loop) diuretics cause loss of potassium as they enable the body to rid itself of excess fluids. Older adults have decreased thirst mechanisms and are at risk for dehydration and subsequent fluid and electrolyte problems. A middle-aged man who is traveling to a hot climate and/or high altitude is at risk for insensible water loss as he acclimates to warmer temperatures but will not necessarily experience electrolyte imbalance. A pregnant patient in the third trimester does have an increase in total body fluids, but this accumulation occurs gradually throughout the pregnancy and should not result in an electrolyte imbalance.

A patient reports swelling of the right foot and ankle. Upon assessing the patient, the health care provider confirms it as pitting edema and prescribes diuretic therapy. Which nursing interventions are necessary for this patient? Select all that apply. a. Monitoring the respiratory rate b. Monitoring the urine output of the patient c. Assessing the sodium and potassium values d. Checking the urine for correct specific gravity e. Monitoring the electrocardiogram patterns (ECG

b, c, e Patients with fluid overload often have pitting edema, and diuretic therapy focuses on removing the excess fluid. The nursing interventions would be monitoring the patient's response to drug therapy, especially increased urine output and weight loss. Diuretic therapy is associated with electrolyte imbalance; therefore sodium and potassium levels need to be monitored. Severe electrolyte disturbances may result in arrhythmias. Therefore changes in the electrocardiogram (ECG) should be monitored. Diuretic therapy does not cause respiratory depression or changes in respiratory rate, so the respiratory rate does not need to be monitored. Checking the urine specific gravity is beneficial in patients to detect the fluid overload. However, it is not useful in patients on diuretic therapy.

A patient reports swelling of the right foot and ankle. Upon assessing the patient, the health care provider confirms it as pitting edema and prescribes diuretic therapy. Which nursing interventions are necessary for this patient? Select all that apply. a. Monitoring the respiratory rate b. Monitoring the urine output of the patient c. Assessing the sodium and potassium values d. Checking the urine for correct specific gravity e. Monitoring the electrocardiogram patterns (ECG)

b, c, e Patients with fluid overload often have pitting edema, and diuretic therapy focuses on removing the excess fluid. The nursing interventions would be monitoring the patient's response to drug therapy, especially increased urine output and weight loss. Diuretic therapy is associated with electrolyte imbalance; therefore sodium and potassium levels need to be monitored. Severe electrolyte disturbances may result in arrhythmias. Therefore changes in the electrocardiogram (ECG) should be monitored. Diuretic therapy does not cause respiratory depression or changes in respiratory rate, so the respiratory rate does not need to be monitored. Checking the urine specific gravity is beneficial in patients to detect the fluid overload. However, it is not useful in patients on diuretic therapy.

A patient is admitted to the nursing unit with a diagnosis of hypokalemia. Which assessment does the nurse complete first? a. Auscultating bowel sounds b. Obtaining a pulse oximetry reading c. Checking deep tendon reflexes (DTRs) d. Determining the level of consciousness (LOC)

b. Because hypokalemia may cause respiratory insufficiency and respiratory arrest, the patient's respiratory status should be assessed first. Bowel sounds, DTRs, and LOC may change in a patient with hypokalemia, but these changes are not immediately life-threatening.

A patient who is suffering from chronic fluid overload asks the nurse to suggest necessary dietary changes. What dietary changes suggested by the nurse apart from restricting fluid intake would be effective to minimize fluid overload? a. Intake of 5 to 6 g/day of sodium b. Intake of 2 to 4 g/day of sodium c. Intake of 3 to 5 g/day of sodium d. Intake of 4 to 6 g/day of sodium

b. Excessive sodium and fluid intake are the main causes of hypervolemia or fluid overload. Nutrition therapy for the patient with fluid overload may involve restriction of sodium and fluid intake. A patient suffering from chronic fluid overload may be restricted to 2 to 4 g/day of sodium. Intake of 5 to 6 g, 3 to 5 g, or 4 to 6 g of sodium per day may lead to further fluid overload and retention.

A patient's morning laboratory results show a serum ionized calcium of 2.85 mmol/L. For what sign must the nurse assess? a. Tachypnea b. Blood clotting c. Muscle spasms d. Increased peristalsis

b. Hypercalcemia allows blood clots to form more easily, especially in the lower legs and pelvic region. The nurse should assess for signs of blood clotting associated with the elevated serum calcium. Increased peristalsis and muscle spasms are associated with hypocalcemia. Tachycardia can occur initially with mild hypercalcemia, but bradycardia is associated with severe hypercalcemia.

When planning care for a patient with hypercalcemia, which intervention does the nurse consider? a. Monitor cardiac rhythm for changes. b. Limit activities to protect against injury. c. Assess oxygen saturation levels every 4 hours. d. Avoid invasive procedures due to increased bleeding tendency.

b. Hypercalcemia increases the risk for cardiac dysrhythmias. It does not impair gas exchange, so oxygen saturation does not need to be routinely monitored. There is a greater tendency to clot, especially with slow venous perfusion, so invasive procedures do not need to be avoided and increased activity (not restriction) is recommended.

The nurse reviews a client's record and determines that the client is at risk for developing a potassium deficit if which situation is documented? a. Sustained tissue damage b. Requires nasogastric suction c.Has a history of Addison's disease d.Uric acid level of 9.4 mg/dL (557 mcmol/L)

b. The normal serum potassium level is 3.5 to 5.0 mEq/L (3.5 to 5.0 mmol/L). A potassium deficit is known as hypokalemia. Potassium-rich gastrointestinal fluids are lost through gastrointestinal suction, placing the client at risk for hypokalemia. The client with tissue damage or Addison's disease and the client with hyperuricemia are at risk for hyperkalemia. The normal uric acid level for a female is 2.7 to 7.3 mg/dL (160 to 430 mcmol/L) and for a male is 4.0 to 8.5 mg/dL (240 to 501 mcmol/L).

A patient with diabetes mellitus who is prescribed diuretic medication reports giddiness and a dry mouth when in dry climates. After assessing the vital signs, the patient is diagnosed as having mild dehydration. Which is the best nursing intervention? a. Discontinue the diuretic drug therapy. b. Consume small amounts of sugar-free fluids hourly. c. Immediately start intravenous fluid replacement therapy. d. Instruct the patient to take oral rehydration solutions containing electrolytes and glucose hourly

b. The patient should be encouraged to drink small volumes of sugar-free fluids hourly. Discontinuing the diuretic drug therapy would be appropriate if the patient is suffering from moderate to severe dehydration but should only be done upon the instructions of the primary health care provider. Intravenous fluid replacement therapy would be appropriate in patients with severe dehydration. The ingestion of oral rehydration solutions containing electrolytes and glucose helps to correct dehydration caused by vomiting and diarrhea.

A patient with Cushing's Syndrome has been experiencing an infection and has a fever of 102'F. On assessment, you find the patient to be confused, restless, has dry mucous membranes, and flushed skin. Which finding below correlates with the presentation of this patient? A. Sodium level of 144 B. Sodium level of 115 C. Sodium level of 170 D. Sodium level of 135

c

The nurse is caring for a client with heart failure. On assessment, the nurse notes that the client is dyspneic, and crackles are audible on auscultation. What additional manifestations would the nurse expect to note in this client if excess fluid volume is present? a. Weight loss and dry skin b. Flat neck and hand veins and decreased urinary output c.An increase in blood pressure and increased respirations d.Weakness and decreased central venous pressure (CVP)

c A fluid volume excess is also known as overhydration or fluid overload and occurs when fluid intake or fluid retention exceeds the fluid needs of the body. Assessment findings associated with fluid volume excess include cough, dyspnea, crackles, tachypnea, tachycardia, elevated blood pressure, bounding pulse, elevated CVP, weight gain, edema, neck and hand vein distention, altered level of consciousness, and decreased hematocrit. Dry skin, flat neck and hand veins, decreased urinary output, and decreased CVP are noted in fluid volume deficit. Weakness can be present in either fluid volume excess or deficit.

Which hormone regulates fluid and electrolyte balance by preventing water and sodium loss? a. Antidiuretic b. Thyrotropin c. Aldosterone d. Natriuretic peptide (NP)

c Aldosterone stimulates the nephrons to reabsorb sodium and water into the blood, preventing sodium and water loss. NP hormone levels are altered in response to increased blood volume and blood pressure. Antidiuretic hormone acts on the collecting ducts of the kidney to normalize the blood osmolarity. Thyrotropin is a thyroid-stimulating hormone that is released in response to low levels of the thyroid hormone.

Which drug therapy might be used to manage symptoms of hypocalcemia? a. Vitamin E b. Zinc sulfate c. Calcium chloride d. Potassium chloride

c Calcium supplements are given to restore serum calcium levels. Potassium, zinc, and vitamin E are not indicated for the management of hypocalcemia.

A patient with a sodium level of 115 is ordered to start Declomycin. Which statement is NOT true about this medication? A. "Declomycin is an antidiuretic hormone antagonist that treats SIADH." B. "Declomycin is part of the tetracycline family." C. "Declomycin is to be taken with food, preferably, milk." D. "Declomycin is contraindicated in children and pregnant/nursing women."

c Declomycin binds to cations (calcium) and this affects GI absorption...it should be given without food.

A patient has edema in both lower extremities. The patient's edema is due to which occurrence? a. Large intercellular spaces b. Filtration of fluid in the cells c. Changes in normal hydrostatic pressure d. Equilibrium in solute concentration on each side of the cell

c Edema develops with changes in normal hydrostatic pressure differences, such as in patients with right-sided heart failure. Large intercellular spaces, filtration of fluid in the cells, and equilibrium in solute concentration on each side of the cell do not affect edema.

The nurse is instructing a patient who is being discharged with a diagnosis of congestive heart failure (CHF). Which patient statement indicates a correct understanding of CHF? a. "I can gain 2 pounds of water a day without risk." b. "I should call my provider if I gain more than 1 pound a week." c. "Weighing myself daily can reveal increased fluid retention." d. "Weighing myself daily can determine if my caloric intake is adequate.

c Fluid retention may not be visible. Rapid weight gain is the best indicator of fluid retention and overload. Each pound of weight gained (after the first half-pound) equates to 500 mL of retained water. The patient should be weighed at the same time every day (before breakfast) on the same scale. Daily weights are not an indication of effective dieting for purposes of weight loss or gain. They will show fluid retention after an especially high sodium intake (in a patient with fluid retention problems), but caloric intake is related to food intake rather than fluid retention problems. The patient should call the health care provider if more than 1 or 2 pounds are gained in a 24-hour period of if more than 3 pounds are gained in 1 week.

A patient with cerebral edema was prescribed a hyperosmotic intravenous (IV) solution. What is the purpose of this intervention? a. Improve diffusion of electrolytes within cells b. Reduce diffusion of potassium into the extracellular space c. Pull excessive water from the interstitial volume to reduce edema d. Reduce filtration of body fluids by increasing hydrostatic pressure in the blood vessels

c Osmosis is the movement of water only through a semipermeable membrane down a concentration gradient. By infusing a hyperosmotic solution, interstitial fluid will be pulled into the plasma volume, reducing cerebral edema. Other physiologic actions, including diffusion and filtration, that help maintain intracellular and extracellular homeostasis are less influenced by the infusion of hyperosmotic IV solution

A patient receiving insulin and glucose infusion therapy for hyperkalemia now has a serum potassium level of 3.6 mEq/L. What is the nurse's first action? a. Stop the infusion immediately. b. Continue the infusion at the prescribed rate. c. Assess the patient's heart rate, rhythm, and respiratory status. d. Slow the infusion and increase the frequency of vital sign assessment.

c The serum potassium is now at the low end of normal range (3.5-5.0 mEq/L). The nurse must first assess the patient's response to the infusion and subsequent change in serum potassium (notably a change in respiratory effectiveness and quality and regularity of the heart rate). Once assessment data are obtained, the nurse should contact the provider and the infusion may be stopped, but it does not have to occur immediately.

A patient with heart failure is hospitalized with shortness of breath and lower extremity pitting edema. Furosemide 5 mg intravenous (IV) push is ordered. Which patient outcome indicates that the medication is effective? a. Weight gain b. Increased heart rate c. Increased urine output d. Decreased blood pressure (BP)

c When giving furosemide, the nurse monitors the patient for response to drug therapy, including improved respiratory status, increased urine output, and a decrease in weight. Although a fall in the patient's BP may occur with the decrease in body fluid, this is not the most important assessment to be monitored. Urinary output is most important. Furosemide may cause a decrease in heart rate as it lowers the patient's body fluid, but this effect would take some time to note. Weight loss, rather than weight gain, is often the effect of furosemide, but it does not occur immediately.

An electrocardiogram (ECG) is ordered for a patient who was placed on intravenous (IV) fluids containing potassium. Which ECG finding is consistent with hyperkalemia? a. Absent T waves b. Elevated P waves c. Prolonged PR intervals d. Shortened QRS complexes

c When hyperkalemia is present, an individual may show absent P waves, tall T waves, prolonged PR intervals, and widened QRS complexes

Which are common symptoms of hypokalemia? Select all that apply. a. Paresthesia b. Bradycardia c. Shallow respirations d. Weak, thready pulse e. Musculoskeletal weaknes

c, d, e Common symptoms of hypokalemia include shallow respirations; weak, thready pulse; and musculoskeletal weakness. Paresthesia and bradycardia are symptoms of hyperkalemia.

The nurse is caring for a patient who is receiving intravenous (IV) magnesium sulfate. Which assessment parameter is critical? a. 24-hour urine output b. Monitoring of serum calcium levels c. Hourly deep tendon reflexes (DTRs) d. Asking the patient about feeling depressed

c. The patient who is receiving IV magnesium sulfate should be assessed for signs of toxicity every hour by assessment of DTRs. Most patients who have fluid and electrolyte problems will be monitored for intake and output (I&O); this will not immediately generate data about problems with magnesium overdose. Low magnesium levels can cause psychological depression, but assessing this parameter as the levels are restored would not be a method by which to safely assess a safe dose or an overdose. Although administration of magnesium sulfate can cause a drop in calcium levels, this occurs over a period of time and would not be the best way to assess magnesium toxicity.

What is the defense mechanism to combat the effects of isotonic dehydration and maintain blood flow to the vital organs? a. Decreased heart rate b. Decreased peripheral resistance c. Increased blood vessel constriction d. Increased pulmonary ventilation rate

c. Dehydration may cause hypovolemia along with decreasing blood pressure. Vasoconstriction is a mechanism that helps to increase the blood pressure and maintain blood flow to the vital organs. The heart rate increases in a hypovolemic condition to maintain the blood flow and blood pressure to the vital organs. Peripheral resistance is increased in order to maintain blood pressure and circulation during dehydration. Increased pulmonary ventilation rate or hyperventilation causes dehydration that further leads to hypovolemia and decreased blood pressure.

What is the reason for isotonic dehydration? a. More sodium then water is lost from the body. b. A decrease in circulating blood volume causes poor tissue perfusion. c. Water and sodium are lost from the body in proportionately equal amounts. d. A shift of fluid from plasma to interstitial space occurs without a loss of any water.

c. Dehydration may occur with either fluid loss or with both fluid and electrolyte loss. Dehydration is termed isotonic dehydration when there is proportionality between the amount of water and electrolyte, such as sodium, lost from the body. Hypertonic dehydration happens when more water than sodium is lost from the body. Hypovolemia occurs when a decrease in the circulating blood volume causes poor tissue perfusion. Relative dehydration occurs without the actual loss of total body water when fluid shifts from plasma to the interstitial space.

What is the reason for relative dehydration? a. Too much fluid loss b. Too little fluid intake c. Fluid shift from plasma to interstitial space d. Fluid shift from interstitial space to plasma

c. Dehydration without actual loss of total body water, such as when the fluid shifts from plasma to the interstitial space, is called relative dehydration. Too much fluid loss is the decrease in the total body water, leading to dehydration. Too little fluid intake also causes an actual decrease in the total water content of the body, which results in dehydration. A fluid shift from the interstitial space to the plasma causes an increase in plasma volume, which is a condition known as hypervolemia.

Which patient is at greatest risk for hypernatremia? a. 30-year-old on a low-salt diet b. 42-year-old receiving hypotonic fluids c. 54-year old who is sweating profusely d. 17-year-old with a serum blood glucose of 189 mg/d

c. Excessive sweating is a common cause of hypernatremia. Hyperglycemia, a low-salt diet, and hypotonic fluid administration are common causes of hyponatremia, not hypernatremia.

Which patient is at greatest risk for hypernatremia? a. 30-year-old on a low-salt diet b. 42-year-old receiving hypotonic fluids c. 54-year old who is sweating profusely d. 17-year-old with a serum blood glucose of 189 mg/dL

c. Excessive sweating is a common cause of hypernatremia. Hyperglycemia, a low-salt diet, and hypotonic fluid administration are common causes of hyponatremia, not hypernatremia.

What statement by the student nurse indicates a need for further teaching about the care to be taken in healthy adults to prevent mild dehydration? a. "Include drinking lots of water in dry climates." b. "Refrain from any beverages containing alcohol." c. "Consume more beverages that contain caffeine." d. "Drink lots of water after prolonged physical activity."

c. Mild dehydration is very common in healthy adults and can be prevented by matching fluid intake with fluid output. Beverages containing caffeine can increase fluid loss, thereby causing more dehydration; therefore beverages containing caffeine should be avoided. Patients who live in dry climates should be advised to drink lots of water. Beverages containing alcohol can increase fluid loss. To balance the fluid lost during physical activity, the patient should take more or equal amount of fluids, preferably water, to prevent dehydration.

What is a consequence of fluid overload? a. Tachycardia b. Hyperthermia c. Pulmonary edema d. Orthostatic hypotension

c. Pulmonary edema may occur due to accumulation of fluid in the air spaces of lungs due to fluid retention. Tachycardia, where the heart rate increases to compensate for decreased fluid loss, is associated with dehydration. Hyperthermia is one of the conditions associated with dehydration. Orthostatic hypotension occurs due to decreased blood pressure, which is a clinical manifestation of dehydration.

An older adult admitted with dehydration and a history of stress incontinence expresses embarrassment about the disorder and the need for absorbent undergarments. Which question about nutritional metabolic needs would be best to ask this patient related to the reason for admission? a. How is your appetite? b. Have you noticed a change in the tightness of your shoes? c. What is your typical urinary elimination pattern and amount? d. What is your typical daily fluid intake and what types of fluids do you drink?

d Asking the patient about the amount and types of fluids will provide additional information as to how the patient deals with incontinence through fluid management and possible causes for the dehydration. Asking the patient about appetite, urinary pattern, and tightness of shoes is not helpful in assessing the patient's current health problem.

A 90-year-old patient with hypermagnesemia is seen in the emergency department (ED). The ED nurse prepares the patient for admission to which inpatient unit? a. Medical-surgical b. Dialysis/home care c. Geriatric/rehabilitation d. Telemetry/cardiac stepdown

d Because hypermagnesemia causes changes in the electrocardiogram that may result in cardiac arrest, the patient should be admitted to the telemetry/cardiac stepdown unit. Dialysis/home care units, geriatric/rehabilitation units, and medical-surgical units typically do not have cardiac monitoring capabilities.

A 77-year-old woman is brought to the emergency department by her family after she has had diarrhea for 3 days. The family tells the nurse that she has not been eating or drinking well, but that she has been taking her diuretics for congestive heart failure (CHF). Her laboratory results include a potassium level of 7.0 mEq/L. What is the primary goal of drug therapy for this patient? a Maintaining proper diuresis and urine output b Elevating serum potassium levels to a safe range c. Decreasing cardiac contractility and slowing the heart rate d. Restoring fluid balance by controlling the causes of dehydration

d Drug therapy for dehydration is directed at restoring fluid balance and controlling the causes of dehydration. Hyperkalemia (serum potassium level of 7.0) will slow the cardiac rate and cause decreased contractility of the heart. Serum potassium levels are already critically high so should not be elevated further. Excessive diuretic use is what has caused this patient's problems. What she needs now is to have electrolyte balance restored; for potassium, that is 3.5 to 5.0 mEq/L.

Which consequence of fluid overload may result in seizures, coma, and death? a. Decreased hematocrit b. Decreased hemoglobin c. Decreased serum proteins d. Decreased serum sodium and potassium levels

d Fluid overload may cause a decrease in serum electrolytes such as sodium and potassium, which can lead to seizures, coma, and death. A decrease in hematocrit due to fluid overload decreases the serum osmolarity, which may cause pulmonary edema or heart failure. A decrease in hemoglobin increases the respiratory rate to meet the oxygen needs of the body. A decrease in serum proteins decreases the serum osmolarity and may cause pulmonary edema or heart failure.

Which nursing action does the nurse include in the care of an alert older adult who is mildly dehydrated? a. Assessing weight and vital signs every 4 hours. b. Restricting oral fluids if the patient is incontinent. c. Advising the patient and family that strict bedrest is recommended due to fall risk. d. Considering dietary restrictions and ability to swallow, and offering oral fluids every 2 hours.

d Oral fluid replacement is a priority when correcting mild to moderate dehydration in an alert patient who can swallow. Because risk for falls is increased, the patient should be offered assistance when ambulating. Bedrest is not recommended and may even contribute to other complications. Oral fluids should not be withheld due to incontinence. Initially, it is recommended to assess vital signs every 2 hours and weigh the patient every 8 hours.

A patient with Crohn's disease reports numbness, tingling, and painful muscle contractions. After assessing the deep tendon reflexes of the patient, which intervention does the nurse perform next? a. Intravenous administration of sodium b. Intravenous administration of bicarbonate c. Intravenous administration of phosphorus d. Intravenous administration of magnesium sulfate

d Patients with Crohn's disease are at a high risk for magnesium imbalance, mainly hypomagnesemia. A decrease in the levels of magnesium may cause increased nerve impulse transmission causing hyperactive deep tendon reflexes, numbness, tingling, and painful muscle contractions. This condition can be well treated by administering magnesium sulfate intravenously. Intravenous administration of sodium, bicarbonate, or phosphorous is not a suitable intervention.

Which assessment finding is consistent with fluid overload? a. Heart murmurs b. Decreased pulse rate c. Decreased respiratory rate d. Moist crackles in the lungs upon auscultation

d Patients with fluid overload will often have moist crackles in the lungs, an increased respiratory rate, and an increased pulse rate. Heart murmurs are not associated with fluid overload.

The nurse is reviewing lab values for a patient recently admitted to the medical-surgical unit. Which lab result is severely abnormal? a. Sodium, 137 mEq/L b. Chloride, 107 mEq/L c. Potassium, 3.5 mEq/L d. Magnesium, 6.2 mEq/L

d. A magnesium level of 6.2 mEq/L is greatly elevated. Patients with severe hypermagnesemia are in grave danger of cardiac arrest. The normal magnesium level is 1.3 to 2.1 mEq/L. The sodium and potassium results are within normal limits. The chloride level is just slightly elevated, with the normal range being between 98 to 106 mEq/L.

Which laboratory value requires a priority response by the nurse to notify the primary health care provider? a. Sodium 133 mEq/L b. Calcium 9.0 mg/dL c. Potassium 5.0 mEq/L d. Magnesium 4.2 mEq/L

d. A magnesium level of 4.2 is markedly elevated (normal 1.3-2.1 mEq/L). Manifestations are not usually apparent until levels exceed 4 mEq/L. Patients with severe hypermagnesemia are in danger of cardiac arrest, so a level of 4.2 mEq/L requires prompt attention. A sodium level of 133 mEq/L is slightly low, but does not require immediate notification of the health care provider. A potassium level of 5.0 mEq/L is normal. A calcium level of 9.0 mg/dL is normal.

What is fluid overload or overhydration? a. Fluid intake is less than body's fluid needs b. Fluid retention is equal to body's fluid needs c. Fluid retention is less than body's fluid needs d. Fluid intake is greater than body's fluid needs

d. Fluid overload or overhydration is an excess of body fluid. It is a condition in which fluid intake or retention is greater than the body's fluid needs. If fluid intake is less than the body's fluid needs, then the condition is called dehydration. Fluid retention is equal to the body's fluid needs in a condition called hydration. If the fluid retention is less than the body's fluid needs, it leads to dehydration.

The nurse instructs an older adult patient to increase intake of dietary potassium when the patient is prescribed which classification of drugs? a. Beta blockers b. Corticosteroids c. Alpha antagonists d. High-ceiling (loop) diuretics

d. High-ceiling (loop) diuretics are potassium-depleting drugs. The patient should increase intake of dietary potassium to compensate for this depletion. Alpha antagonists, beta blockers, and corticosteroids are not potassium-depleting drugs.

A patient is brought to the emergency department with symptoms of diarrhea, chest discomfort, and paresthesia. The patient has a heart rate of 60 beats per minute. The electrocardiogram (ECG) of the patient shows missed P waves, tall T waves, prolonged PR intervals, and wide QRS complexes. Which laboratory finding would be consistent with the patient's condition? a. Serum sodium levels below 135 mEq/L b. Serum sodium levels above 145 mEq/L c. Serum potassium levels below 3.5 mEq/L d. Serum potassium levels above 5.0 mEq/L

d. Laboratory data confirms hyperkalemia if serum potassium levels are above 5.0 mEq/L. Hyperkalemia clinically manifests with cardiovascular changes such as bradycardia, hypotension, and ECG changes that include missed P waves, tall T waves, prolonged PR intervals, and wide QRS complexes. A neuromuscular change associated with hyperkalemia is numbness around the mouth, or paresthesia. Intestinal changes include frequent and watery bowel movements. Abnormalities in sodium levels are not associated with hyperkalemia. Hypokalemia is associated with serum potassium levels below 3.5 mEq/L, which do not show any such ECG changes.

What is the major cause of death in patients diagnosed with hypokalemia? a. Stroke b. Renal failure c. Cardiac arrest d. Respiratory insufficiency

d. Respiratory changes may occur in patients with hypokalemia because of respiratory muscle weakness resulting in shallow respirations. The respiratory status of a patient with hypokalemia should be assessed at least every 2 hours because respiratory insufficiency is the major cause of death for these patients. A stroke is not a risk factor for hypokalemia. Hypokalemia does not cause renal failure; rather, hyperkalemia is caused by renal failure. Dysrhythmias may occur due to hypokalemia but are not the major cause of death in patients with hypokalemia.


Related study sets

7. Cloud Application Security (This one was hard) 80%

View Set

Chapter 14-Infection, Infectious Diseases & Epidemiology

View Set

Art and myth of the classical world study guide

View Set